Sie sind auf Seite 1von 100

www.insightsonindia.

com www.insightsias.com
TEST 9 Solutions

Prelims 2017 Test 9


SOLUTIONS

1. Gandhiji was invited to the inauguration of the Banaras Hindu


University. His speech there is considered important from many
viewpoints. The speech included which of the following?
1. He criticized the use of English in the forum.
2. He admonished the Maharajas for stealing and concentrating
wealth away from the poor.
3. He asked the attendee Viceroy to leave India and return back to
England.

Select the correct answer using the codes below.

a) 1 only
b) 2 and 3 only
c) 1 and 2 only
d) 1, 2 and 3

Solution: d)
Justification: In 1916, less than a year after Gandhis arrival in India,
he was invited by Annie Besant for the inauguration ceremony of
Banaras Hindu University.

Gandhi delivered a sharply-worded speech at the event in the


presence of the Viceroy, Lord Harding, a galaxy of maharajas and
the top educationists of the time.
He sharply criticized the use of English, the filth that was allowed
to gather around Indias temples, the civic sense of railway
travellers, and admonished the maharajas present for the jewels
they were wearing.
He accused them of stealing wealth from the poor farmers and, to
cap it all, even blamed the chief guest, Lord Harding, for creating a
wall of distrust by surrounding himself with such heavy security.


INSIGHTS PRELIMS TEST SERIES 2017

www.insightsonindia.com www.insightsias.com
TEST 9 Solutions

The final straw was when he asked the Viceroy to go home to


England. This was so unexpected and outlandish that his
outspokenness evoked disfavour.
Annie Besant, seated on the dais, shouted at Gandhi asking him to
stop, while the maharajas left in disgust. The event had to be called
off to avoid further embarrassment to the dignitaries, but the
audience seemed appreciative of Gandhis approach. The gentry in
India were indeed discovering Gandhi.

Q Source: Improvisation: CAPF past year papers

2. Consider the following about Ganeshwar-Jodhpura culture?


1. It is associated with the lineage of the rural elites of the Ahars.
2. The culture is known for the practice of copper metallurgy.

Which of the above is/are correct?

a) 1 only
b) 2 only
c) Both 1 and 2
d) None

Solution: b)
Justification: Statement 1 and 2: Ahars were a separate culture from
GJs. Located in northeast Rajasthan, the Ganeshwar-Jodhpura complex
was an early centre of agriculture and copper metallurgy in the
subcontinent. More than eighty sites of this culture have been identified
in the Sikar, Jaipur and Jhunjhunu districts, with the largest
concentration being in Sikar.

It is possible that the inhabitants of Khetri (GJs) region supplied


copper to the Harappans.
Among the earliest chalcolithic cultures in India, the Ahar or Banas
culture was discovered in the Mewar region of southeast
Rajasthan.
Nearly one hundred sites of the culture have been located along its
principal axis, i.e., the valleys of river Banas and its tributaries and


INSIGHTS PRELIMS TEST SERIES 2017

www.insightsonindia.com www.insightsias.com
TEST 9 Solutions

subtributaries in Banswara, Udaipur etc. The technology at Ahar


was based mainly on copper and very few microblades and
microliths have been discovered.
You can read more here
http://www.academia.edu/1813929/Indices_of_Interaction_Com
parisons_between_the_Ahar-
Banas_and_Ganeshwar_Jodhpura_Cultural_Complex

Q Source: Improvisation: Page 12: Themes in Indian History I

3. Which of the following were the pillars of the Nehru - Mahalanobis


strategy of development?
1. High consumption rate to support consumer goods industries
2. Policies that promote competition of domestic firms with foreign
firms
3. Import substitution
4. Capitalistic pattern of society

Select the correct answer using the codes below.

a) 1 and 2 only
b) 3 only
c) 2 and 4 only
d) 1, 2 and 3 only

Solution: b)
Justification: Statement 1: The major pillars were - (a) high savings
rate: It was belived that high saings rate would help finance large
investment in heavy industries. When savings are high consumption
cant be high, so 1 is wrong.

Statement 2: Other pillars were heavy industry bias and protectionist


policies and public sector. If protection was not granted to the weak
Indian domestic industries, they may not have survived against strong
foreign competition. So, 2 is also wrong.


INSIGHTS PRELIMS TEST SERIES 2017

www.insightsonindia.com www.insightsias.com
TEST 9 Solutions

Statement 3: Import substitution was favoured due to Indias harsh


colonial experience with foreign and the need to develop an indigenous
base of capital goods and other industries. So, 3 is correct.

Statement 4: Nehru was a socialist, so 4 cant be correct.

Q Source: Improvisation: Page 20: Indian Economic Development:


Class XIth NCERT

4. Consider the following about the Privilege motion in Parliament.


1. It can be admitted only in Lok Sabha.
2. It is always raised against either the entire council of Ministers or
the House.
3. All matters related to the motion must be referred to the privilege
committee of the Lok Sabha.
4. It can be raised by a private member.

Select the correct answer using the codes below.

a) 2 and 3 only
b) 4 only
c) 1 and 4 only
d) 1 and 3 only

Solution: b)
Justification: Recently, opposition member in Lok Sabha has given a
notice to move privilege motion against Human Resource Development
minister.

Notice contended that Human Resource Development minister has


misled the House and the Nation on the issue of government
intervention in Delhis JNU and the Hyderabad Central University while
intervening in a debate.

Statement 2: Privilege Motion is moved by a member when he feels that


a minister or any member has committed a breach of privilege of the


INSIGHTS PRELIMS TEST SERIES 2017

www.insightsonindia.com www.insightsias.com
TEST 9 Solutions

House or one or more of its members by withholding facts of a case or by


giving wrong or distorted facts.

Its purpose is to censure the concerned minister or any other member.

Statement 1: Each of the two houses, the Lok Sabha and the Rajya
Sabha, has separate privilege committees, made up of their members. It
can be introduced in both.

Statement 3: The presiding officers of the two Houses, the Speaker and
the Chairperson respectively, can dismiss privilege notices, or refer them
to the privilege committee, or get a sense of the House before taking a
decision

Statement 4: Privilege motion can be moved by any lawmaker/MP


against anyone accused of breaching parliamentarians' privileges, their
special rights and immunities.

Q Source: http://www.thehindu.com/news/national/opposition-ups-
ante-to-bring-privilege-motion-against-smriti-irani/article8289898.ece

5. Consider the following about TRIPS +.


1. It is an outcome of the Nairobi package at the WTO.
2. It is concerned with enhanced IPR protections in party nations.
3. India has ratified TRIPS+ agreement.

Select the correct answer using the codes below.

a) 1 and 3 only
b) 2 only
c) 2 and 3 only
d) 1, 2 and 3

Solution: b)
Justification: The TRIPS Agreement allows Members to have higher
levels of protection than the minimum standards laid down in it, thus


INSIGHTS PRELIMS TEST SERIES 2017

www.insightsonindia.com www.insightsias.com
TEST 9 Solutions

leaving the flexibility to Members to have TRIPS plus laws and


regulations.

The developed countries are moving toward higher, enhanced


standards of IPR protection to evolve TRIPS-plus regime.
These higher standards are now making an appearance in various
free trade agreements (FTA) that these countries are negotiating
and entering into with their trading partners.

Statement 1: It was not a part of the Nairobi package, and instead being
negotiated in regional FTAs.

Statement 2 and 3: India is opposed to TRIPS+ provisions. India is an


IPR consumer unlike developed nations who are IPR producers.

India has not reached a high level of IPR implementation such as


Japan and the EU and therefore it now favours only a TRIPS
Agreement-like IPR chapter in CEPAs.
Since TRIPS+ provisions go beyond minimum standards
established under TRIPS, they may take away the flexibilities (for
example the ability to issue compulsory licenses for medicines
required in public health emergencies) that exist in the TRIPS
Agreement.
Inclusion of TRIPS-plus' provisions in Comprehensive Economic
Partnership Agreements (CEPA) would restrict generics drugs
exports from India and prevent Indian companies from obtaining
the latest technology at affordable prices from abroad, they said.

Q Source: Often in news


INSIGHTS PRELIMS TEST SERIES 2017

www.insightsonindia.com www.insightsias.com
TEST 9 Solutions

6. What was\were the fundamental difference(s) in the philosophies


of Lokayatas and Ajivikas?
1. Ajivikas believed in fatalism, whereas Lokayatas believed the
opposite.
2. Ajivikas noted that there are no Universal laws, whereas
Lokayatas believed certain fundamental laws existed.

Which of the above is/are correct?

a) 1 only
b) 2 only
c) Both 1 and 2
d) None

Solution: a)
Justification: Statement 1: Fatalists or Ajivikas were those who believe
that everything is predetermined. Materialists or Lokayatas were those
who believe that everything is not predetermined.

Statement 2: Fatalists argue that the wise and the fool cannot come out
of karma. It can neither be lessened nor increased. So fool and wise alike
will take their course and make an end of sorrow. This is because the
Universe operates on certain fundamental laws, and the human being is
no exception to those laws.

Materialists say that a human being is made up of the four elements.


When he dies the earthy in him returns to the earth, the fluid to water,
the heat to fire, the windy to air, and his senses pass into space. The talk
of gifts is a doctrine of fools, an empty lie. Fools and wise alike are cut off
and perish. They do not survive after death.

Q Source: Page 87: Themes in Indian History I


INSIGHTS PRELIMS TEST SERIES 2017

www.insightsonindia.com www.insightsias.com
TEST 9 Solutions

7. Consider the following with reference to the views of the French


traveller, Bernier about India.
1. He wrote that Sanskrit was closest to European languages and
sponsored several works of inter-translation of texts.
2. He denounced the system of crown ownership of land.
3. He was associated with the Mughal court as an economic and
social advisor.

Select the correct answer using the codes below.

a) 1 and 2 only
b) 2 and 3 only
c) 2 only
d) 1 and 3 only

Solution: c)
Justification:Statement 1: This was Al-Birunis view who visited India
much earlier. Bernier visited India in the 17th Century.

Statement 2: As per Bernier, one of the fundamental differences


between Mughal India and Europe was the lack of private property in
land and crown ownership of land as being harmful for both the state
and its people.

He thought that in the Mughal Empire the emperor owned all the
land and distributed it among his nobles, and nobles to the
peasants.
Owing to crown ownership of land, landholders could not pass on
their land to their children. So they were averse to any long-term
investment in the sustenance and expansion of production.

Statement 3: He was a doctor, political philosopher and historian. He


came to the Mughal Empire in search of opportunities.

He was closely associated with the Mughal court, as a physician to Prince


Dara Shukoh, the eldest son of Emperor Shah Jahan, and later as an
intellectual and scientist, with Danishmand Khan, an Armenian noble at
the Mughal court.


INSIGHTS PRELIMS TEST SERIES 2017

www.insightsonindia.com www.insightsias.com
TEST 9 Solutions

Learning: Bernier travelled to several parts of the country, and wrote


accounts of what he saw, frequently comparing what he saw in India with
the situation in Europe.

He found Bengal as prosperous as Egypt and found it a major exporter of


many agricultural commodities.

Q Source: Improvisation: Page 5: Indian Economic Development: Class


XIth NCERT

8. Commercialisation of agriculture was one of the major policies of


the British Empire in rural India. What do you understand by this
term?
1. Land reforms for greater revenue generation
2. Agricultural production for sale in the market
3. Making tenants the sole owner of the land

Select the correct answer using the codes below.

a) 1 and 2 only
b) 2 only
c) 3 only
d) 1 and 3 only

Solution: b)
Justification: Commercialization can be defined as the process
wherein farmers start producing primarily for sale in distant markets
rather than for localized consumption or for sale in domestic markets
(distant doesnt necessarily mean exports; long-distance in-country
trade, made possible by railways, also counts).

The first wave of commercial agriculture was driven largely by Indigo


and Opium.

Statement 1: Land reforms, though separate from commercialization of


agriculture, propelled commercialization. Burdened with high land rents,


INSIGHTS PRELIMS TEST SERIES 2017

www.insightsonindia.com www.insightsias.com
TEST 9 Solutions

farmers had no escape route than to grow agricultural produce for sale in
the market.

Statement 3: British adopted policies that made them the ultimate


owners of land. Under different systems, different stakeholders were
made only the leaseholders of the land such as Zamindar.

Q Source: Improvisation: Page 6:Indian Economic Development: Class


XIth NCERT

9. The Ross Sea was recently declared as worlds largest Marine


Protected Area (MPA) to protect the Earths most pristine marine
ecosystem. It is located in the
a) Southern Ocean
b) Arctic Sea
c) North Atlantic Sea
d) Sargasso Sea

Solution: a)
Learning: The Ross Sea is one of the last intact marine ecosystems in
the world. It covers 1.6 million square kilometers.

Ross Sea in the Southern Ocean is home to 38% of the worlds


Adelie penguins, 30% of worlds Antarctic petrels and around 6%
of worlds population of Antarctic minke whales. It is also home to
huge numbers of krill, a staple food for species including whales
and seals.

It was declared a MPA after landmark international agreement


among 24 countries and European Union in Commission for the
Conservation of Antarctic Marine Living Resources (CCAMLR).

MPA status to Ross Sea will bring blanket ban on commercial


fishing across about three quarters of its area for 35 years.


INSIGHTS PRELIMS TEST SERIES 2017

www.insightsonindia.com www.insightsias.com
TEST 9 Solutions

Q Source: http://www.thehindu.com/sci-tech/energy-and-
environment/countries-ok-worlds-largest-marine-reserve-in-
antarctica/article9279727.ece

10. The Union Cabinet has recently approved the establishment


and operationalisation of a National Academic Depository (NAD).
Consider the following about it.
1. It will host all international journals on science and humanities
under one database.
2. It will be a single window mechanism for stakeholders to enquire
about Centrally administered Universities and colleges.


INSIGHTS PRELIMS TEST SERIES 2017

www.insightsonindia.com www.insightsias.com
TEST 9 Solutions

Which of the above is/are correct?

a) 1 only
b) 2 only
c) Both 1 and 2
d) None

Solution: d)
Justification: None of the above are correct as NAD has been
established with different objectives.

The Finance Ministers Budget Speech (2016) had incorporated


commitment to establish a Digital Depository for school learning
certificates, degrees and other academic awards of Higher
Education Institutions, on the pattern of a Securities Depository.

Under it all academic degrees, certificates and awards in the


country will be made digitally available for verification on single
spot.

Academic institutions will be directed to upload and authenticate


all documents in digital form into the NAD system

Requests for access to academic awards will be on basis of consent


of the owner. For ex, access from potential employers and
academic institutions will be only on the basis of consent of the
student. NAD will maintain the authenticity, integrity and
confidentiality of its database. It will root out fake degrees.

Q Source: http://pib.nic.in/newsite/PrintRelease.aspx?relid=153035

11. One of the most important characteristic of Indias foreign trade


throughout the colonial period was the generation of a large export
surplus. This was achieved largely by
a) Exploitative colonial policies that used the surplus to fund their
trade
b) Near zero duties on cash crops and textile industry in India


INSIGHTS PRELIMS TEST SERIES 2017

www.insightsonindia.com www.insightsias.com
TEST 9 Solutions

c) Policy of discriminating protection granted to select industries


since the 19th Century
d) Growing share of industrial activity during the colonial regime

Solution: a)
Justification: After the Battle of Plassey in 1757, the English East India
Company began converting a large portion of the tax revenue from
conquered areas into funds for investments.

These investments were then used to purchase Indian goods


(from Indian money), and these goods were then sold
internationally. This represented a wholesale transfer of Indian
revenues to the Companys coffers in Britain
This meant that Britain did not need to export anything to India in
return for what it obtained from India as imports.
Thus, there arose a large excess of Indian exports over imports
(export surplus - around 4% of national income) pretty much all
the way through before the WW1. Even in face of ever-increasing
British imports (textiles etc.), India almost consistently maintained
an export surplus of over 20% over imports till 1914; but this didnt
translate into any benefit for the economy.

Learning: This surplus came at a huge cost to the countrys economy.


Several essential commoditiesfood grains, clothes, kerosene etc.
became conspicuous by their acute scarcity in the domestic market.
Furthermore, this export surplus did not result in any flow of gold or
silver into India.

Rather, this was used to make payments for the expenses incurred
by an office set up by the colonial government in Britain, expenses
on war, again fought by the British government, and the import of
invisible items, all of which led to the drain of Indian wealth.

Q Source: Improvisation: Page :Indian Economic Development: Class


XIth NCERT


INSIGHTS PRELIMS TEST SERIES 2017

www.insightsonindia.com www.insightsias.com
TEST 9 Solutions

12.The Suez Canal provides a direct trade route for ships operating
between
1. European ports to South Asian ports
2. American ports to West African ports
3. Chinese ports to South Asian ports

Select the correct answer using the codes below.

a) 1 and 2 only
b) 2 and 3 only
c) 1 only
d) 1, 2 and 3

Solution: c)
Justification:

As you can see in the figure, Chinese route to South Asian route does not
require the use of Suez Canal. So, 3 is wrong.

American ports to West African ports can be accessed directly via


Atlantic Ocean. So, 2 is wrong too.

Q Source: Improvisation: Page 9: Indian Economic Development: Class


XIth NCERT


INSIGHTS PRELIMS TEST SERIES 2017

www.insightsonindia.com www.insightsias.com
TEST 9 Solutions

13. Replacement fertility rates often heard in news are those rates at
which
a) Maternal Mortality Rate (MMR) would be constant
b) Birth rates would become zero
c) Population would stabilize
d) Infant Mortality Rate (IMR) would become zero

Solution: c)
Learning: Total fertility rate (TPFR) of a population is the average
number of children that would be born to a woman over her lifetime
under certain conditions (including her survival).

Replacement level fertility is the total fertility rate at which women


give birth to enough babies to sustain population levels in the
nation.
At this rate death rates become equal to the birth rates and
population growth stops, i.e. population is constant throughout
and does not decline. This is crucial for a stable demography and
reducing population pressure.
At this rate a population will exactly replace itself from one
generation to the next, without migration. This rate is roughly 2.1
children per woman for most countries, although it may modestly
vary with mortality rates and be higher for some developing
countries.
In India, Kerala has achieved below replacement level fertility rates
at around 1.8, which means its population will see a decline in the
coming decades.

Q Source: Improvisation: Page 10: Indian Economic Development:


Class XIth NCERT


INSIGHTS PRELIMS TEST SERIES 2017

www.insightsonindia.com www.insightsias.com
TEST 9 Solutions

14.In a socialist economic system


1. Market forces of supply and demand do not exist
2. Only those goods are produced which yield sustained profits
3. The government guides the overall economic activity in the
economy

Select the correct answer using the codes below.

a) 1 only
b) 1 and 2 only
c) 3 only
d) 1, 2 and 3

Solution: c)
Justification: Statement 1: The market forces of supply and demand
will exist universally as long as some goods are traded (supplied) and
some are purchased (demanded). Even if the government guides the
economic activity, the provision of certain goods will create supply-
demand mismatches and the government will feel the heat (if not in the
form of prices for goods that come under administered prices regime).
So, 1 is incorrect as it is an extreme statement.

Statement 2: This is the guiding principle begin a purely capitalistic


system.

Statement 3: In a socialist society the government decides what goods


are to be produced in accordance with the needs of society. It is assumed
that the government knows what is good for the people of the country
and so the desires of individual consumers are not given much
importance.

Q Source: Page 18: Indian Economic Development: Class XIth NCERT


INSIGHTS PRELIMS TEST SERIES 2017

www.insightsonindia.com www.insightsias.com
TEST 9 Solutions

15. Early Sangam texts mention different categories of people.


Consider the following matches of these categories with their social
status.
1. Uzhavar : Zamindars
2. Vellalar : Local banker
3. Adimai : Slaves

Select the correct answer using the codes below.

a) 1 and 2 only
b) 3 only
c) 1 and 3 only
d) 2 only

Solution: b)
Justification: There was a growing differentiation amongst people
engaged in agriculture were based on differential access to land, labour
and some of the new technologies.

Early Sangam texts mention different categories of people engaged


in agriculture were based on differential access to land, labour and
some of the new technologies.

In south India people who were divided into three groups large
landowners or vellalar, ploughmen or uzhavar and slaves or
adimai.

In the country side of Northern India people were divided into


three groups - landless agricultural labourers, small peasants, as
well as large landholders.

The term gahapati was often used in Pali texts to designate the
second and third categories.

Q Source: Page 39: Themes in Indian History I


INSIGHTS PRELIMS TEST SERIES 2017

www.insightsonindia.com www.insightsias.com
TEST 9 Solutions

16.The Delimitation Act, 2002 act and Representation of the People


Act, 1950 were amended recently to take into account
a) Increased population since 2011 Census
b) Rotation of new constituencies between male and female
candidates
c) India Bangladesh land boundary agreement
d) Establishment of State Election Commissions in states where
they didnt exist earlier

Solution: c)
Justification: The Union Cabinet recently gave its approval to amend
section 11 of the Delimitation Act, 2002 and section 9 of the
Representation of the People Act, 1950.

It will enable Election Commission to carry out limited delimitation of


Assembly and Parliamentary Constituencies in the Cooch Behar District
of West Bengal consequent upon the exchange of 51 Bangladeshi
enclaves and 111 Indian enclaves respectively between India and
Bangladesh.

Learning: Under Article 82 of the Constitution, the Parliament by law


enacts a Delimitation Act after every census.

After coming into force commencement of the Act, the Central


Government constitutes a Delimitation Commission.

This Delimitation Commission demarcates the boundaries of the


Parliamentary Constituencies as per provisions of the Delimitation
Act.

The present delimitation of constituencies has been done on the


basis of 2001 census figures under the provisions of Delimitation
Act, 2002.

Q Source: PIB: Amendment of these acts


INSIGHTS PRELIMS TEST SERIES 2017

www.insightsonindia.com www.insightsias.com
TEST 9 Solutions

17. Consider the following statements.

Assertion (A): Subsidies do not allow prices to indicate the supply


of a good and prevent the market from fixing the price of
subsidized goods.

Reason (R): All the goods and services subsidized by the


government come under the administered price regime.

In the context of the above, which of these is correct?

a) A is correct, and R is an appropriate explanation of A.


b) A is correct, but R is not an appropriate explanation of A.
c) A is correct, but R is incorrect.
d) Both A and R are incorrect.

Solution: d)
Justification: Assertion has two parts:

One, that subsidies do not allow prices to indicate the supply of a


good. When electricity and water are provided at a subsidised rate
or free, they will be used wastefully without any concern for their
scarcity (low supply - which is indeed the case).If the price was
instead higher, consumption would be prudent and value the
goods scarcity. So, the first part of the argument is correct.
Second part of the argument is that subsidies prevent the market
from fixing the price of subsidized goods. This is not true.
Many services such as kerosene, fertilizers, educational services,
LPG etc are subsidized by the government; yet the market is free to
determine their price. It is a different matter what the consumer
pays in the end, but the fact that the actual price is decided by the
market is correct. So, Second part of the argument is wrong, and so
A is wrong.

Reason (R ) is absurd, as commodities like education services and LPG


do not come under the administered price regime i.e. their prices are not
fixed at a certain level by the government.


INSIGHTS PRELIMS TEST SERIES 2017

www.insightsonindia.com www.insightsias.com
TEST 9 Solutions

Q Source: Improvisation: Page 27: Indian Economic Development:


Class XIth NCERT

18. According to Sanskrit legal texts, women were not supposed


to have independent access to resources such as land. But, this
women ruler who was associated with the Gupta Empire had
access to land and is renowned as one of the most important rulers
in early Indian history. She is?
a) Nilambhaprada
b) Prabhavati
c) Lalleshwari
d) Chowteshwari

Solution: b)
Justification: Prabhavati Gupta was the daughter of Chandragupta II.

Women were not supposed to have independent access to


resources such as land in those times.

However, the inscription indicates that Prabhavati had access to


land, which she then granted.

This may have been because she was a queen and her situation was
therefore exceptional.

Learning: There were regional variations in the sizes of land donated


ranging from small plots to vast stretches of uncultivated land to donees
(the recipients of the grant).

Some Historians feel that land grants were part of a strategy adopted by
ruling lineages to extend agriculture to new areas.

Others suggest that when kings were losing control over their samantas,
they tried to win allies by donating lands to people.

Q Source: Page 40: Themes in Indian History I


INSIGHTS PRELIMS TEST SERIES 2017

www.insightsonindia.com www.insightsias.com
TEST 9 Solutions

19.Consider the following with reference to NITI Aayog.


1. It hosts the Team India Hub which leads the engagement of
states with the Central government.
2. It acts as a repository of research on good governance best
practices.
3. It has been nominated as the nodal agency for bringing the
Sustainable Development Goals (SDGs) into action across India.

Select the correct answer using the codes below.

a) 1 only
b) 2 and 3 only
c) 1 and 2 only
d) 1, 2 and 3

Solution: d)
Justification: Statement 1: At the core of NITI Aayogs creation are
two hubs Team India Hub and the Knowledge and Innovation Hub.
The Team India Hub leads the engagement of states with the Central
government, while the Knowledge and Innovation Hub builds NITIs
think-tank capabilities. These hubs reflect the two key tasks of the Aayog.

Statement 2: NITI Aayog is also developing itself as a State of the Art


Resource Centre, with the necessary resources, knowledge and skills,
that will enable it to act with speed, promote research and innovation,
provide strategic policy vision for the government, and deal with
contingent issues.

Statement 3: NITI Aayog will monitor, coordinate and ensure


implementation of the globally accepted Sustainable Development Goals.
We have been nominated as the nodal body that will bring the 17
development goals into action across India. The SDGs are aimed at
eradicating all forms of poverty.


INSIGHTS PRELIMS TEST SERIES 2017

www.insightsonindia.com www.insightsias.com
TEST 9 Solutions

Q Source: Improvisation: Chapter 2: Planning in India: Indian


Economic Development: Class XIth NCERT

20. James Webb Space Telescope (JWST) will be useful to


understand which of the following?
1. Origins and evolution of our solar system
2. Signs of life on faraway planets

Which of the above is/are correct?

a) 1 only
b) 2 only
c) Both 1 and 2
d) None

Solution: c)
Justification & Learning: JWST is the largest-ever space telescope is
expected to launch in 2018.

It is the formal successor to the Hubble Space Telescope and the


Spitzer Space Telescope. After its launch it will be the premier
observatory of the next decade.

It has a total area of 270 square feet. Hubbles mirror has an area
of just 48 square feet. Thus, JWST will be able to see much fainter
objects than Hubble.

It will have cameras and spectrometers that are able to record


extremely faint signals.

It will help in broad range of investigations across the fields of


astronomy and cosmology. It will help to understand the origins of
the universe, evolution of our own Solar System, search for signs of
life on faraway planets. It can also analyze the atmospheres of
exoplanets that pass in front of their stars


INSIGHTS PRELIMS TEST SERIES 2017

www.insightsonindia.com www.insightsias.com
TEST 9 Solutions

Q Source: www.jwst.nasa.gov/

21.When government expenditure is more than its receipts


1. It floats rupee in the international market.
2. It purchases sovereign bonds of credit rated countries.
3. It increases the amount of Current Account deficit to fund the
fiscal deficit.
4. It purchases securities under the Market Stabilisation Scheme
(MSS).

Select the correct answer using the codes below.

a) 2 and 3 only
b) 4 only
c) 1, 2 and 4 only
d) None of the above

Solution: d)
Justification: Statement 1: Rupee already floats (to an extent under
our managed float system) in the forex market. Floating rupee does not
help the government in meeting its fiscal deficit. So,1 is wrong.

Statement 2: The government should sell such bonds so that buyers give
money to the government for it to meet its fiscal deficit. Purchase makes
matters worse. So, 2 is wrong.

Statement 3: High CAD may further aggravate governments fiscal


deficit, as it may lead to higher inflation in the economy and thus a
further pressure on government finances. So, 3 is wrong too.

Statement 4: Under the MSS( Market Stabilisation Scheme) securities


are issued with the objective of providing the RBI with a stock of
securities with which it can intervene in the market for managing
liquidity. The government does not intervene in the market with MSS.


INSIGHTS PRELIMS TEST SERIES 2017

www.insightsonindia.com www.insightsias.com
TEST 9 Solutions

So, 4 is wrong. Moreover, as said earlier, sale of such securities is


required, not their purchase.

Q Source: Improvisation: Page 39: Indian Economic Development:


Class XIth NCERT

22. Development defects and diseases are increasingly becoming


the focus of government health policies. Given below are some of
these diseases. Which of the following is/are caused by genetic
disorders?
1. Turners Syndrome
2. Downs Syndrome
3. Lenders Mutadrome

Select the correct answer using the codes below.

a) 1 only
b) 1 and 2 only
c) 2 and 3 only
d) 1, 2 and 3

Solution: b)
Justification: Statement 1: Such a disorder is caused due to the
absence of one of the X chromosomes. Such females are sterile as ovaries
are rudimentary besides other features including lack of other secondary
sexual characters.

Statement 2: The cause of this genetic disorder is the presence of an


additional copy of the chromosome number 21. The affected individual is
short statured with small round head, furrowed tongue and partially
open mouth. Mental development is retarded.

Q Source: Page 91: 12th Biology NCERT


INSIGHTS PRELIMS TEST SERIES 2017

www.insightsonindia.com www.insightsias.com
TEST 9 Solutions

23. According to the recent data given by the Registrar General


of India (RGI), the state with the highest life expectancy in India
is?
a) Tamil Nadu
b) Arunachal Pradesh
c) Jammu & Kashmir (J&K)
d) Punjab

Solution: c)
Learning: Jammu and Kashmir has the highest life expectancy (i.e.
post-60 life expectancy), except life expectancy at birth

Jammu and Kashmir has surpassed Kerala which used to be the


leader in life expectancy in almost all categories of age till 2010.

However, Kerala still has the highest overall life expectancy at


birth at 74.9 years.

RGIs Sample Registration System (SRS) had published state level


life expectancy data prepared following surveys done between 2010
and 2014.

It had defined life expectancy as estimate of average number of


additional years that a person of a given age can expect to live.

Q Source: http://www.deccanherald.com/content/577882/life-
expectancy-highest-jammu-kashmir.html


INSIGHTS PRELIMS TEST SERIES 2017

www.insightsonindia.com www.insightsias.com
TEST 9 Solutions

24. Consider the following about the recently launched Biotech-


KISAN Scheme.
1. Fellowship will be given to women farmers for training and
education in farm practice.
2. Government will undertake an ambitious project of genome
sequencing of registered indigenous cattle breeds of India.

Which of the above is/are correct?

a) 1 only
b) 2 only
c) Both 1 and 2
d) None

Solution: a)
Justification: Statement 1: Its purpose is to connect farmers, scientist
and science institution across country.

Under it, fellowship will be given to women farmers for training and
education in farm practice.

Scientists will spend time on farms and link communication tools to soil,
water seed and market.

The main aim of the scheme is to understand individual problems of the


small holding farmers and provide ready solutions.

Statement 2: This is a part of another scheme that was launched


simultaneously Cattle Genomics.

The Cattle Genomics Scheme aims at boosting selective breeding of the


native livestock more accurately to ensure high-yielding, disease-
resistant, resilient livestock.

Q Source: http://pib.nic.in/newsite/PrintRelease.aspx?relid=151986


INSIGHTS PRELIMS TEST SERIES 2017

www.insightsonindia.com www.insightsias.com
TEST 9 Solutions

25. Which of the following measures may lead to an increase in


the inflow of foreign exchange in India?
1. Devaluation of rupee
2. Stable and low inflation in India

Which of the above is/are correct?

a) 1 only
b) 2 only
c) Both 1 and 2
d) None

Solution: c)
Justification: Statement 1: Devaluation attracts foreing investors as
now they get more Indian assets by investing less in dollar or other
foreign currency terms.

For e.g. in 1991, as an immediate measure to resolve the balance of


payments crisis, the rupee was devalued against foreign currencies.
This led to an increase in the inflow of foreign exchange.
However, if devaluation is made arbitrary and volatile, there may
actually be a considerable outflow of foreign exchange on account
of an uncertain investment climate, as happened during the 2008
recession.

Statement 2: Inflation shows that demand of goods is higher than


supply, i.e. if companies invest (FDI) their goods will be picked up by the
consumers.

Very high inflation discourages investment due to its uncertain


nature and high drag on wage costs of firms.
Stable Inflation makes investment profitability predictable and
thus encourages FDI.

Q Source: Improvisation: Page 42: Indian Economic Development:


Class XIth NCERT


INSIGHTS PRELIMS TEST SERIES 2017

www.insightsonindia.com www.insightsias.com
TEST 9 Solutions

26. The TRIMS Agreement sometimes seen in news is related


to
1. Domestic regulations over Foreign direct investment (FDI)
2. Current Account Convertibility
3. Trade balancing Requirements of developing countries

Select the correct answer using the codes below.

a) 2 only
b) 1 and 3 only
c) 1 and 2 only
d) 3 only

Solution: b)
Justification: In the late 1980s, there was a significant increase
in foreign direct investment throughout the world.

However, some of the countries receiving foreign investment imposed


numerous restrictions on that investment designed to protect and foster
domestic industries, and to prevent the outflow of foreign exchange
reserves.

These Conditions which can affect trade are known as trade-related


investment measures or TRIMs of WTO.

Statement 1 and 3: Examples of these restrictions include local content


requirements (which require that locally produced goods be purchased
or used), manufacturing requirements (which require the domestic
manufacturing of certain components), trade balancing requirements
(i.e. stepping up import duty to arrest imports in view of a high trade
deficit), domestic sales requirements, technology transfer requirements,
export performance requirements (which require the export of a
specified percentage of production volume) etc.

The present TRIMS agreement prohibits trade related investment


measures that are violative of Trade balancing Requirements and Export
Restrictions of the General Agreement on Tariffs and Trade. So, Local


INSIGHTS PRELIMS TEST SERIES 2017

www.insightsonindia.com www.insightsias.com
TEST 9 Solutions

content requirements, trade balancing requirements, and export


restrictions are prohibited.

India is a signatory to TRIMS and has fulfilled major obligations related


to the agreement.

Statement 2: CAC means liberalizing the conversion of rupee into


foreign currencies and vice versa for meeting out current account
transactions, such as exports-imports. India has already liberalized CAC
and thus CAC is not relevant to the TRIMS. 2 is wrong.

Learning: India was dragged to the WTO dispute resolution


mechanism by the US in 2013, regarding Indias domestic content
requirements under the Jawaharlal Nehru National Solar Mission, for
solar cells and solar modules; the US claims that this amounts to
preferential treatment to national industry, and hence violates GATT and
TRIMS.

Q Source: Current Affairs + Improvisation: Page 43: Indian Economic


Development: Class XIth NCERT

27. Consider the following about the Inter-State Council.


1. It is not a permanent constitutional body.
2. It is chaired by the Prime Minister.
3. It does not include the representatives of the Union Territories
(UTs).
4. It includes the Presiding officers of both houses of Parliament.

Select the correct answer using the codes below.

a) 1 and 2 only
b) 2 and 4 only
c) 1 and 3 only
d) 2, 3 and 4 only

Solution: a)


INSIGHTS PRELIMS TEST SERIES 2017

www.insightsonindia.com www.insightsias.com
TEST 9 Solutions

Justification: It is a recommendatory body to investigate and discuss


subjects, in which some or all of the states or the Central government
have a common interest. It is set up on the basis of provisions in Article
263 of the Constitution of India.

Statement 1: The Inter-state council is not a permanent constitutional


body for coordination between the states and Central government.
Rather, President can establish it at any time if it appears to him that the
public interests would be served by the establishment of such a council.

Statement 2, 3 and 4: It facilitates coordination between states and the


centre. It is headed by the Prime Minister. Composition of reconstituted
Inter State Council (ISC) Members: Chief Ministers of all states and
Union Territories (having legislative Assembly), Six Union Ministers.
Besides, administrators of Union Territories which do not have
legislative Assembly are also members of the Council.

Q Source: Re-constituted Inter-State Council recently

28. In India, which of these states/UTs are well known for the
production of oil seeds in India?
1. Chandigarh
2. Rajasthan
3. Tamil Nadu
4. Madhya Pradesh
5. Andaman and Nicobar Islands

Select the correct answer using the codes below.

a) 2 and 4 only
b) 2, 3 and 4 only
c) 1, 3 and 5 only
d) 1, 2, 3, 4 and 5

Solution: b)


INSIGHTS PRELIMS TEST SERIES 2017

www.insightsonindia.com www.insightsias.com
TEST 9 Solutions

Justification: Statement 1 and 5: Most UTs do not produce oilseeds


like Chandigarh, A&N islands, Daman Diu. Some others like Delhi and
Goa contribute very little to production.

Statement 2: MP, Maharashtra, AP, Gujarat, Rajasthan, UP and TN are


some of the major producer of oilseeds in India.

For e.g. Tamil Nadu is the second largest producer accounting for over
22 per cent of the total groundnut produced in India.

Q Source: Improvisation: Page 49: Indian Economic Development:


Class XIth NCERT

29. The poverty gap index measures


a) The proportion of population that is multi-dimensionally poor
b) The proportion of population that would be below the poverty
line without positive state intervention
c) The proportion of population that is not employed in full-time
regular jobs
d) None of the above

Solution: d)
Learning: There are many ways to measure poverty.


INSIGHTS PRELIMS TEST SERIES 2017

www.insightsonindia.com www.insightsias.com
TEST 9 Solutions

The headcount index measures the proportion of the population that is


poor. It is popular because it is easy to understand and measure. But it
does not indicate how poor the poor are. The poverty gap index
measures the extent to which individuals fall below the poverty line (the
poverty gaps) as a proportion of the poverty line.

The sum of these poverty gaps gives the minimum cost of eliminating
poverty, if transfers were perfectly targeted. The measure does not reflect
changes in inequality among the poor. The squared poverty gap index
(also known as the poverty severity index) averages the squares of the
poverty gaps relative to the poverty line. It

Q Source: Improvisation: Page 65: Indian Economic Development:


Class XIth NCERT

30. The Chipko movement was launched against


a) Untouchability
b) Environmental conservation
c) Globalization
d) Commercialization of even basic services

Solution: b)
Learning: In the 1970s, an organized resistance to the destruction of
forests spread throughout India and came to be known as the Chipko
movement. The name of the movement comes from the word 'embrace',
as the villagers hugged the trees, and prevented the contractors' from
felling them.

With encouragement from a local NGO (non-governmental


organization), DGSS (Dasoli Gram Swarajya Sangh), the women of the
area, under the leadership of an activist, Chandi Prasad Bhatt, went into
the forest and formed a circle around the trees preventing the men from
cutting them down.


INSIGHTS PRELIMS TEST SERIES 2017

www.insightsonindia.com www.insightsias.com
TEST 9 Solutions

It went on to become a rallying point for many future environmental


movements all over the world; it created a precedent for non-violent
protest started in India.

Q Source: Indian Economic Development: 11th NCERT

31. Consider the following statements with reference to the economic


statistics over the past decade in India.
1. The labour force participation rate for women has increased.
2. The size of the organized sector has increased positively.

Select the correct answer using the codes below.

a) 1 only
b) 2 only
c) Both 1 and 2
d) None

Solution: a)
Justification: Statement 1: Female participation rates have declined by
nearly 9-10% over the last decade largely due to improvement in
economic status of families, movement of women to education (such as
higher education), lack of acceptable jobs in rural areas which are
transitioning as urban zones etc.

Statement 2: Over the past two decades, the proportion of organized


sector has grown in India by around 1% each decade. This is despite the
fact that the new employment generated is largely in the informal sector,
e.g. construction workers.

Q Source: Improvisation: Chapter 7: Indian Economic Development:


Class XIth NCERT


INSIGHTS PRELIMS TEST SERIES 2017

www.insightsonindia.com www.insightsias.com
TEST 9 Solutions

32. Mangrove forests have reached a critical stage of decline in


many parts of the world. Which of these have caused the gravest
threat to mangroves?
1. Destruction of coral reefs
2. Fresh water diversion away from seas and oceans due to
hydraulic structures

Which of the above is/are correct?

a) 1 only
b) 2 only
c) Both 1 and 2
d) None

Solution: c)
Justification: Statement 1: Coral reefs provide the first barrier against
currents and strong waves. When they are destroyed, the stronger-than-
normal waves and currents reaching the coast can undermine the fine
sediment in which the mangroves grow. This can prevent seedlings from
taking root and wash away nutrients essential for mangrove ecosystems.

Statement 2: River changes are caused due to dams and irrigation


reducing the amount of water reaching mangrove forests. This changes
the salinity level of water in the forest. If salinity becomes too high, the
mangroves cannot survive. Freshwater diversions can also lead to
mangroves drying out.

Oil pollution has also led to the threat; it can smother mangrove roots
and suffocate the trees.

You can read more here


http://wwf.panda.org/about_our_earth/blue_planet/coasts/mangroves
/mangrove_threats/

Q Source: Revision previous test syllabus: Fundamentals of Physical


Geography: 11th NCERT


INSIGHTS PRELIMS TEST SERIES 2017

www.insightsonindia.com www.insightsias.com
TEST 9 Solutions

33. The royal title of Devaputra and Hindu Suratrana are


often associated with which of these ruler/kingdoms respectively?
a) Kushanas and Vijayanagara
b) Magadha and Samudragupta
c) Shakas and Cholas
d) Vakatakas and Cheras

Solution: a)
Learning: One means of claiming high status was to identify with a
variety of deities. This strategy is best exemplified by the Kushanas (c.
first century BCE-first century CE), who ruled over a vast kingdom
extending from Central Asia to northwest India.

Colossal statues of Kushana rulers have been found installed in a


shrine at Mat near Mathura (Uttar Pradesh). Similar statues have
been found in a shrine in Afghanistan as well. Some historians feel
this indicates that the Kushanas considered themselves godlike. Many
Kushana rulers also adopted the title devaputra, or son of god,
possibly inspired by Chinese rulers who called themselves sons of
heaven.

The Vijayanagara kings claimed to rule on behalf of the god


Virupaksha. Rulers also indicated their close links with the gods by
using the title Hindu Suratrana this meant Hindu Sultan.

Q Source: Improvisation: Page 36: Themes in Indian History I

34. Special purpose vehicles (SPVs) often seen in news are


related to
a) Underwater observatories in Arctic
b) Military operations
c) Wilful defaults on bank loans
d) Project investment and execution

Solution: d)


INSIGHTS PRELIMS TEST SERIES 2017

www.insightsonindia.com www.insightsias.com
TEST 9 Solutions

Learning: Special purpose vehicles (SPV) are often used to fund


projects in the infrastructure sector.

Some lend funds, especially debt funds of longer maturity, directly to


eligible projects to supplement loans from banks and financial
institutions. Some help in executing projects, such as SPVs formed under
JNUURM earlier and Smart cities project now.

SPVs may be formed through limited partnerships, trusts, corporations

Q Source: Improvisation: Page 143: Indian Economic Development:


Class XIth NCERT

35. The replacement of CFLs with LED lights is likely to be


ecologically beneficial because CFLs contain this element and
LEDs do not. It is?
a) Mercury
b) Lead
c) Nickel
d) Copper

Solution: a)
Learning: Older CFLs tend to contain higher amounts of mercury. A
CFL is a sealed unit, and no mercury is released when it is in use or as
long as it is intact. Some mercury is released when a bulb breaks

LEDs use significantly less energy than even CFLs, and do not contain
mercury, phosphorous and sulphur.

But LEDs do have a dark side. A study published in late 2010 in the
journal Environmental Science and Technology found that LEDs contain
lead, arsenic and a dozen other potentially dangerous substances like
nickel. It also contains copper.

Q Source: Improvisation: Page 148: Indian Economic Development:


Class XIth NCERT


INSIGHTS PRELIMS TEST SERIES 2017

www.insightsonindia.com www.insightsias.com
TEST 9 Solutions

36. Consider the following about rural Public health


infrastructure.
1. Primary health care Centres (PHCs) must compulsorily be
located in every village.
2. Auxiliary Nursing Midwife (ANM) is the first person who
provides primary healthcare in rural areas.
3. PHCs are capable of handling institutional deliveries.

Select the correct answer using the codes below.

a) 1 and 2 only
b) 2 and 3 only
c) 3 only
d) 1, 2 and 3 only

Solution: b)
Justification: Statement 1 and 2: Sub-Centre (SC) is the most
peripheral and first contact point between the primary health care
system and the community. One Sub Centre is to cover a population of
3000 in Hilly / Tribal / Difficult areas and 5000 in Plain areas. These
generally cover one or more villages. So, 1 is wrong, as it is a sub-centre
rather than a PHC.

One PHC is to cover a population of 20000 in Hilly/ Tribal / Difficult


areas and 30000 in Plain areas.

Statement 2: Each Sub-Centre is required to be manned by at least one


Auxiliary Nurse Midwife (ANM) / Female Health Worker and one Male
Health Worker who is the first contact point for the administration of
primary health care in these areas.

Statement 3: You can see it here http://www.thehindu.com/todays-


paper/tp-national/tp-tamilnadu/institutional-deliveries-on-the-rise-in-
phcs/article1975364.ece and here


INSIGHTS PRELIMS TEST SERIES 2017

www.insightsonindia.com www.insightsias.com
TEST 9 Solutions

http://www.livemint.com/Opinion/55N9wi8w2SO1lEE9MHPvaL/Dista
nce-and-institutional-deliveries-in-rural-India.html

You can read more here https://data.gov.in/catalog/number-sub-


centres-primary-health-centres-and-community-health-centres-
established-during

Q Source: Page 152: Indian Economic Development: Class XIth NCERT

37. Which of these were identified as Harappan sites located in


India?
1. Sutkagendor
2. Mitathal
3. Balakot
4. Dholavira
5. Kalibangan

Select the correct answer using the codes below.

a) 1, 4 and 5 only
b) 1, 2 and 3 only
c) 2, 4 and 5 only
d) 1, 2, 3, 4 and 5

Solution: c)
Justification:Kalibangan: In the mature-Harappan stage, great cities
emerged. The excavations at Kalibangan with its elaborate town
planning and urban features prove this phase of evolution.

There was another important region. Lothal remained an emporium of


trade between the Harappan civilization and the remaining part of India
as well as Mesopotamia.


INSIGHTS PRELIMS TEST SERIES 2017

www.insightsonindia.com www.insightsias.com
TEST 9 Solutions

Q Source: Page 2: Themes in Indian History I

38. Global Youth Development Index (YDI) is published by


a) Commonwealth Secretariat
b) United Nations Development Programme (UNDP)
c) Carnegie Melon Institute
d) International Bureau of Education (IBE)

Solution: a)


INSIGHTS PRELIMS TEST SERIES 2017

www.insightsonindia.com www.insightsias.com
TEST 9 Solutions

Justification & Learning: India has been ranked 133rd out of 183
countries in the 2016 Global Youth Development Index (YDI) compiled
by the Commonwealth Secretariat.

The Global Youth Development Index report measures countries


prospects for young people in employment, education, health, civic
and political spheres.

Top countries in 2016 YDI are Germany (1), Denmark (2),


Australia (3).

Indias neighbours: Sri Lanka (31), Bhutan (69) and Nepal (77).

India accounts for nearly 20% of the global youth population, with
nearly 345 million young people between the ages of 15 and 30
living in the country.

Indias overall rank in 2016 YDU is relatively low, but it has


registered an 11% improvement in scores over last five years (from
2010 and 2015).

The report highlights need for greater investment to reap so-called


demographic dividend. Youth development levels in India lag
particularly in the domains of education, health and employment.

Q Source: http://indianexpress.com/article/india/india-news-
india/india-ranks-poorly-at-133-in-global-youth-development-index-
compiled-by-commonwealth-secretariat-3727154/

39. The terms Absorptive Capacity and Carrying capacity are


used in the discourse of
a) Evolutionary biology
b) Geographical determinism
c) Employment elasticity
d) Sustainable Development

Solution: d)


INSIGHTS PRELIMS TEST SERIES 2017

www.insightsonindia.com www.insightsias.com
TEST 9 Solutions

Learning: This is straight from the NCERT.

The environment is able to perform these functions without any


interruption as long as the demand on these functions is within its
carrying capacity.

This implies that the resource extraction is not above the rate of
regeneration of the resource and the wastes generated are within the
assimilating capacity of the environment.

When this is not so, the environment fails to perform its third and vital
function of life sustenance and this results in an environmental crisis.

Q Source: Improvisation: Page 164: Indian Economic Development:


Class XIth NCERT

40. Consider the following about External Commercial


Borrowings (ECBs).
1. It forms a part of Indias capital account.
2. It can be used by start-ups operating from India.
3. It can be used to finance only Equity transactions of companies.

Select the correct answer using the codes below.

a) 1 and 2 only
b) 2 and 3 only
c) 1 only
d) 1, 2 and 3

Solution: a)
Justification: Statement 1: ECBs are defined as money borrowed from
foreign resources including the following: (i) Commercial bank loans (ii)
Buyers credit and suppliers credit (iii) Credit from official export credit
agencies and commercial borrowings from the private sector window of
Multilateral Financial Institutions such as World Bank, ADB, AFIC,
CDC, etc.


INSIGHTS PRELIMS TEST SERIES 2017

www.insightsonindia.com www.insightsias.com
TEST 9 Solutions

Since they form a part of our capital account, they add to Indias external
debt.

Statement 2: The Reserve Bank of India (RBI) has permitted startups to


raise external commercial borrowings (ECBs) of up to $3 million in a
financial year for three year tenure The new rules issued by RBI aims at
boosting innovation and promoting job creation in the country.

Statement 3: In case of Foreign Direct Investment, the foreign money is


used to finance the Equity Capital. But in case ECBs, foreign money is
used to finance any kind of funding other than Equity.

Under the ECB route, borrowing of startups should be denominated in


any freely convertible currency or in Indian Rupees (INR) or a
combination thereof.

Q Source: http://economictimes.indiatimes.com/small-biz/money/rbi-
allows-start-ups-to-raise-3-million-through-
ecbs/articleshow/55098181.cms

41.Which of these factors are responsible for the problem of land


degradation in India?
1. Forest fires
2. Improper irrigation use in farms
3. Intensive extraction of ground water
4. Monoculture crops

Select the correct answer using the codes below.

a) 2 and 4 only
b) 1 and 3 only
c) 1, 2 and 4 only
d) 1, 2, 3 and 4

Solution: d)


INSIGHTS PRELIMS TEST SERIES 2017

www.insightsonindia.com www.insightsias.com
TEST 9 Solutions

Justification: Statement 1: Forest fires lead to loss of vegetation, top


soil cover, crucial biodiversity like microorganisms and animals thereby
degrading the natural balance of the ecosystem, and leads to soil
degradation.

Statement 2 and 3: Intensive irrigation of farms depletes groundwater;


causes hardening of soil due to higher salinity and water logging leading
to soil degradation.

Statement 4: Practice of monoculture does not allow the soil to rotate


and conserve crucial minerals as the same minerals and nutrients are
depleted in each crop cycle.

Learning: Some other reasons for land degradation are:(i) loss of


vegetation occurring due to deforestation (ii) unsustainable fuel wood
and fodder extraction (iii) shifting cultivation (iv) encroachment into
forest lands (v) over grazing (vi) non-adoption of adequate soil
conservation measures (vii) improper crop rotation (viii) indiscriminate
use of agro-chemicals such as fertilisers and pesticides (ix) improper
planning and management of irrigation systems. Finally human
settlements and industries exert an enormous pressure on the countrys
finite land resources.

The per capita forestland in the country is only 0.08 hectare against the
requirement of 0.47 hectare to meet basic needs, resulting in an excess
felling of about 15 million cubic metre forests over the permissible limit.

Q Source: Page 169: Indian Economic Development: Class XIth NCERT

42. The sixth century BCE is regarded as a major turning point in


early Indian history. It is associated with which of the following?
1. Use of silver for the first time
2. Development of coinage
3. Practice of gana oligarchies in mahajanpadas

Select the correct answer using the codes below.

a) 2 only


INSIGHTS PRELIMS TEST SERIES 2017

www.insightsonindia.com www.insightsias.com
TEST 9 Solutions

b) 1 and 3 only
c) 1 and 2 only
d) 2 and 3 only

Solution: d)
Justification: Statement 1 and 2: Silver was being used since Harappan
times. So, 1 is wrong. It is an era associated with early states, cities, the
growing use of iron, the development of coinage, etc.

Statement 3: It also witnessed the growth of diverse systems of thought,


including Buddhism and Jainism. Early Buddhist and Jaina texts
mention, amongst other things, sixteen states known as mahajanapadas.

While most mahajanapadas were ruled by kings, some, known as ganas


or sanghas, were oligarchies, where power was shared by a number of
men, often collectively called rajas. Both Mahavira and the Buddha
belonged to such ganas.

In some instances, as in the case of the Vajji sangha, the rajas probably
controlled resources such as land collectively.

Q Source: Page 29: Themes in Indian History I

43. Sinai Peninsula lies between


a) Mediterranean Sea and Red Sea
b) Red Sea and Black Sea
c) Arabian Sea and Gulf of Aden
d) Gulf of Aden and Red Sea

Solution: a)


INSIGHTS PRELIMS TEST SERIES 2017

www.insightsonindia.com www.insightsias.com
TEST 9 Solutions

Learning: It is situated between the Mediterranean Sea to the north


and the Red Sea to the south, serving as a land bridge between Asia and
Africa. It is the only part of Egyptian territory located in Asia.

Israel invaded and occupied Sinai during the Suez Crisis of 1956,
and during the Six-Day War of 1967. In 1973, Egypt launched the
Yom Kippur War to retake the peninsula, which was the site of
fierce fighting between Egyptian and Israeli forces.
By 1982, as a result of the Israel-Egypt Peace Treaty of 1979, Israel
had withdrawn from all of the Sinai Peninsula except the
contentious territory of Taba, which was returned after a ruling by
a commission of arbitration in 1989.

Q Source: Map-based questions: Africa

44. In vitro fertilisation is similar to


1. Test tube baby programme
2. Surrogacy
3. Reverse sterilization


INSIGHTS PRELIMS TEST SERIES 2017

www.insightsonindia.com www.insightsias.com
TEST 9 Solutions

Select the correct answer using the codes below.

a) 1 only
b) 1 and 3 only
c) 2 and 3 only
d) 1, 2 and 3 only

Solution: a)
Justification: Couples could be assisted to have children through
certain special techniques commonly known as assisted reproductive
technologies (ART).

In vitro fertilisation (IVFfertilisation outside the body in almost similar


conditions as that in the body) followed by embryo transfer (ET) is one
of such methods.

In this method, popularly known as test tube baby programme, ova from
the wife/donor (female) and sperms from the husband/donor (male) are
collected and are induced to form zygote under simulated conditions in
the laboratory.

The zygote or early embryos could then be transferred into the mother.

Q Source: Page 64: 12th Biology NCERT

45. Consider the following about the Commission for the


Conservation of Antarctic Marine Living Resources (CCAMLR).
1. It is a part of the Antarctic Treaty System.
2. It was established to protect krill resources which are a keystone
component of the Antarctic ecosystem.

Which of the above is/are correct?

a) 1 only
b) 2 only
c) Both 1 and 2


INSIGHTS PRELIMS TEST SERIES 2017

www.insightsonindia.com www.insightsias.com
TEST 9 Solutions

d) None

Solution: c)
Justification: The Commission for the Conservation of Antarctic
Marine Living Resources (CCAMLR) was established by international
convention in 1982 with the objective of conserving Antarctic marine life.

This was in response to increasing commercial interest in Antarctic krill


resources, a keystone component of the Antarctic ecosystem and a
history of over-exploitation of several other marine resources in the
Southern Ocean.

CCAMLR is an international commission with 25 Members, and a


further 11 countries have acceded to the Convention (including India).

Q Source: http://www.thehindu.com/sci-tech/energy-and-
environment/countries-ok-worlds-largest-marine-reserve-in-
antarctica/article9279727.ece

46. What is the correct North-to-South order of these major


ports of Africa and adjoining countries?
1. Port of Durban
2. Port Elizabeth
3. Port Louis
4. Port of Lisbon

Select the correct answer using the codes below.

a) 4312
b) 2314
c) 3241
d) 1423

Solution: a)


INSIGHTS PRELIMS TEST SERIES 2017

www.insightsonindia.com www.insightsias.com
TEST 9 Solutions

Justification: Eliminate options.

If you know that Lisbon is in Portugal which is up north of Africa, all


options other than A can be eliminated.

If you didnt know that, Port Louis is in Mauritius which is up north of


South Africa (where Durban and Elizabeth ports are located). This
means options B and D could be eliminated.

Q Source: Map-based questions: Africa

47. Consider the following statements.


1. Mauryas used punch-marked coins made of silver and copper.
2. The first coins to bear the names and images of rulers were
issued by the Indo-Greeks.
3. Yaudheyas of Punjab and Haryana were renowned for issuing
copper coins.
4. Both Kushanas and Guptas issued gold coins.

Select the correct answer using the codes below.

a) 1 and 2 only


INSIGHTS PRELIMS TEST SERIES 2017

www.insightsonindia.com www.insightsias.com
TEST 9 Solutions

b) 2, 3 and 4 only
c) 1, 3 and 4 only
d) 1, 2, 3 and 4

Solution: d)
Justification & Learning: Exchanges were facilitated by the
introduction of coinage. Punch-marked coins made of silver and copper
(c. sixth century BCE onwards) were amongst the earliest to be minted
and used by many dynasties including the Mauryas.

The first coins to bear the names and images of rulers were issued
by the Indo-Greeks, who established control over the north-
western part of the subcontinent c. second century BCE.

The first gold coins were issued in the first century CE by the
Kushanas (some sources dispute this fact). These were virtually
identical in weight with those issued by Roman emperors and the
Parthian rulers of Iran.

Coins were also issued by tribal republics of Punjab and Haryana


called the Yaudheyas. Archaeologists have unearthed several
thousand copper coins issued by the Yaudheyas.

Some of the most spectacular gold coins were issued by the Gupta
rulers. The earliest coins issued by Guptas were remarkable for
their purity. These coins facilitated long-distance transactions from
which kings also benefited.

Q Source: Page 44: Themes in Indian History I

48. Todas and Moplahs tribes can be found in the states of


a) Rajasthan and Nagaland
b) Tamil Nadu and Kerala
c) West Bengal and Odisha


INSIGHTS PRELIMS TEST SERIES 2017

www.insightsonindia.com www.insightsias.com
TEST 9 Solutions

d) Andhra Pradesh and Tamil Nadu

Solution: b)
Learning: The Toda people are a small pastoral tribal community who
live on the isolated Nilgiri plateau in hill country of Southern India.

Since the early 21st century, Toda society and culture have been
the focus of an international effort at culturally sensitive
environmental restoration.
The Toda lands are now a part of the Nilgiri Biosphere Reserve, a
UNESCO-designated International Biosphere Reserve; their
territory is declared UNESCO World Heritage Site.
The Moplah peasant movement was engineered in August 1921
among the peasants of Malabar district in Kerala. The Moplah
tenants were Muslims and they agitated against the Hindu
landlords and the British government.
Their grievances related to lack of any security of tenure, renewal
fees, high rents and other oppressive landlord exactions. In the
19th century as well, there had been cases of Moplah resistance to
landlord oppression but what erupted in 1921 was on a different
scale altogether.

Q Source: Tribes of India

49. Consider the following about the Pradhan Mantri Awas


Yojana.
1. The scheme will be functional only in rural areas.
2. The scheme targets Below Poverty Line (BPL) citizens only.

Which of the above is/are correct?

a) 1 only
b) 2 only


INSIGHTS PRELIMS TEST SERIES 2017

www.insightsonindia.com www.insightsias.com
TEST 9 Solutions

c) Both 1 and 2
d) None

Solution: d)
Justification: Union Government had launched Pradhan Mantri Awas
Yojana in 2015 in line with its ambitious Housing for All by 2022
Mission.

Under this scheme government plans to build two crore houses for
the poor in urban areas by year 2022, which coincides with 75
years of Independence. So, 1 is wrong.
The scheme has four basic components viz. slum rehabilitation,
affordable housing, credit linked interest subsidy and beneficiary-
led individual house construction or enhancement.
Intended beneficiary will be poor people (BPL) and people living
under EWS and LIG categories in urban establishments of the
country. Under this scheme, the ownership of houses will be in the
name of woman or jointly with husband.

Q Source: http://www.businesstoday.in/current/economy-
politics/pm-narendra-modi-awas-yojana-five-things-to-
know/story/239225.html

50. Which of the following practices were associated with the


Harappan Settlements?
1. Ploughing of crop fields
2. Consumption of animal products
3. Mixed cropping
4. Practice of Irrigation

Select the correct answer using the codes below.

a) 1 and 4 only
b) 1, 2 and 4 only
c) 2 and 3 only
d) 1, 2, 3 and 4


INSIGHTS PRELIMS TEST SERIES 2017

www.insightsonindia.com www.insightsias.com
TEST 9 Solutions

Solution: d)
Justification: Statement 2: The Harappans ate a wide range of animal
products. Archaeologists have been able to reconstruct use of animals
from finds of charred animal bones found at Harappan sites. These
include those of cattle, sheep,goat, buffalo and pig.

Statement 1 and 3: Archaeologists have found evidence of a ploughed


field at Kalibangan (Rajasthan).The field had two sets of furrows at right
angles to each other, suggesting that two different crops were grown
together.

Representations on seals and terracotta sculpture indicate that the bull


was known, and oxen were used for ploughing.

Terracotta models of the plough have been found at sites in Cholistan


and at Banawali (Haryana).

Statement 4: Most Harappan sites are located in semi-aridlands, where


irrigation was probably required for agriculture. Traces of canals, water
reservoirs and wells have been found at the Harappan sites indicate that
agriculture was practiced.

Q Source: Chapter 1: Themes in Indian History I

51. Consider the following about the CORAL project of NASA.


1. It will evaluate the state of existing rainforests and their
evolutionary future discounting for climate change.
2. It will survey keystone species in the harshest climates on earth
which are the edge of a Goldilocks zone.
3. It will assess the conditions of the major reefs of the world
through remote-sensing.

Select the correct answer using the codes below.

a) 1 and 2 only
b) 1 only


INSIGHTS PRELIMS TEST SERIES 2017

www.insightsonindia.com www.insightsias.com
TEST 9 Solutions

c) 3 only
d) 2 and 3 only

Solution: c)
Justification: NASA has embarked on an air-borne three-year field
experiment called the Coral Reef Airborne Laboratory (CORAL), which
aims to survey the conditions of the major reefs of the world through
remote-sensing.

It will survey the condition of entire reef systems in Hawaii, Palau, the
Mariana Islands and Australia.

CORAL will involve the analysis of this data using underwater


photographs and reef primary productivity.

Learning: Importance of Coral Reefs:

Protect shorelines by absorbing the wave energy. The survival of


many island is dependent on them
Controlling the CO2 in the ocean water
Best breeding ground for fishes
Promotes Eco-tourism
Q Source: http://www.nasa.gov/image-feature/jpl/nasa-s-coral-to-
study-the-condition-of-earth-s-coral-reefs

52. Consider the following statements.


1. Harappans were aware of the decimal system and used it in
assigning denominations of weights.
2. Harappans had a script of their own which remains undeciphered
to date.

Which of the above is/are correct?

a) 1 only
b) 2 only
c) Both 1 and 2


INSIGHTS PRELIMS TEST SERIES 2017

www.insightsonindia.com www.insightsias.com
TEST 9 Solutions

d) None

Solution: c)
Justification: Statement 1: Exchanges were regulated by a precise
system of weights, usually made of a stone called chert and generally
cubical with no markings.

The lower denominations of weights were binary (1, 2, 4, 8, 16, 32, etc).
While the higher denominations followed the decimal system, the
smaller weights were probably used for weighing jewellery and beads
and bigger weights were used for food grains.

Statement 2: Harappan seals usually have a line of writing probably


containing the name and title of the owner.

It is apparent that the script was written from right to left as some seals
show a wider spacing on the right and cramping on the left, as if the
engraver began working from the right and then ran out of space.

A variety of objects on which writing has been found: seals, copper tools
etc.

Q Source: Page 15: Themes in Indian History I

53. Archaeological Survey of India (ASI) is Indias premier


agency for the conservation and discovery of artefacts. Who was
the first Director-General of ASI?
a) John Marshall
b) Alexander Cunningham
c) R.E.M. Wheeler
d) Rakhal Das Banerji

Solution: b)
Learning: Cunningham, the first Director-General of the ASI, began
archaeological excavations in the mid-nineteenth century. Cunninghams
main interest was in the archaeology of the Early Historic (c. sixth
century BCE-fourth century CE) and later periods.


INSIGHTS PRELIMS TEST SERIES 2017

www.insightsonindia.com www.insightsias.com
TEST 9 Solutions

He used the accounts left by Chinese Buddhist pilgrims who had


visited the sub-continent between the fourth and seventh centuries
CE to locate early settlements.
John Marshall, another Director-General of ASI marked a major
change in Indian archaeology. He was the first professional
archaeologist to work in India, and brought his experience of
working in Greece and Crete to the field.
As S.N. Roy noted in The Story of Indian Archaeology, Marshall
left India three thousand years older than he had found her.This
was because similar, till-then-unidentified seals were found at
excavations at Mesopotamian sites. It was then that the world
knew not only of a new civilisation, but also of one
contemporaneous with Mesopotamia.
Q Source: Page 6: Themes in Indian History I

54. Which of these regions is the most visited global medical


tourism destination in India?
a) Chennai
b) Kolkata
c) Ahmadabad
d) Indore

Solution: a)
Learning: The cost-consciousness factor and availability of accredited
facilities have led to emergence of several global medical tourism
corridors - Singapore, Thailand, India, Malaysia, Taiwan, Mexico and
Costa Rica.

Amongst these corridors of health, India has the second largest


number of accredited facilities (after Thailand).


INSIGHTS PRELIMS TEST SERIES 2017

www.insightsonindia.com www.insightsias.com
TEST 9 Solutions

Chennai, Mumbai, AP and NCR are the most favoured medical


tourism destinations for the floating medical population who avail
treatments in India. Kerala is still catching up with these centres.
The city of Chennai has been termed "India's health capital".
Chennai attracts about 45 percent of health tourists from abroad
arriving in the country and 30 to 40 percent of domestic health
tourists. Factors behind the tourists inflow in the city include low
costs, little to no waiting period and facilities offered at the
specialty hospitals in the city.
Q Source: Improvisation: Page 153: Indian Economic Development:
Class XIth NCERT

55. Which of these was/were some of the most important ideas


in Jainism?
1. The entire world is fundamentally inanimate despite the contrary
illusion.
2. There are no cycles of birth and death as objects come from
nothingness and dissolve into nothingness.

Which of the above is/are correct?

a) 1 only
b) 2 only
c) Both 1 and 2
d) None

Solution: d)
Justification: Statement 1: The most important idea in Jainism is that
the entire world is animated: even stones, rocks and water have life.
Non-injury to living beings, especially to humans, animals, plants and
insects, is central to Jaina philosophy.

Statement 2: According to Jaina teachings, the cycle of birth and rebirth


is shaped through karma. Asceticism and penance are required to free


INSIGHTS PRELIMS TEST SERIES 2017

www.insightsonindia.com www.insightsias.com
TEST 9 Solutions

oneself from the cycle of karma. This can be achieved only by renouncing
the world.

Q Source: Page 88: Themes in Indian History I

56. The Saur Sujala Yojana intends to provide


a) Establishment of smart grid infrastructure in rural areas
b) Micro-irrigation facility on every farm in India
c) Solar rooftop panels mandatorily on each house
d) Solar powered irrigation pumps to farmers at a subsidized price

Solution: d)
Learning: It was launched in Chhattisgarh as part of the states 16th
foundation day celebrations.

With this Chhattisgarh becomes the first state to implement the scheme.

Under the scheme, solar powered irrigation pumps will be distributed to


the farmers.

The beneficiaries would get the pumps at the subsidised price.

It is mainly aimed to give priority to those areas where electricity has not
reached, so that farmers can utilize them in irrigation and agriculture.

Around 51,000 farmers would be benefitted in the state with the launch
of the scheme by 2018.

Q Source: http://pmjandhanyojana.co.in/saur-sujala-yojana-scheme/

57. The international group New Agenda for Coalition (NAC)


sometimes seen in news


INSIGHTS PRELIMS TEST SERIES 2017

www.insightsonindia.com www.insightsias.com
TEST 9 Solutions

a) Promotes the interest of Least Development Countries (LDCs) in


getting greater export market share in developed countries
b) Vouches for climate change financing from developed countries
for developing countries
c) Promotes the Non-Proliferation Treaty (NPT) and pushes for
nuclear disarmament worldwide
d) Advocates global economic governance reforms in the World
Bank and IMF

Solution: c)
Learning: NAC is a geographically dispersed group of middle power
countries that promotes the NPT and pushes for nuclear disarmament
worldwide.

It consists of Brazil, Egypt, Ireland, Mexico, New Zealand and South


Africa.

The group was officially launched in Dublin (Ireland) in 1998 in


response to the North-South divide that stymied talks on nuclear
disarmament and non-proliferation within the framework of the NPT.

During the recent visit of New Zealand PM, India failed to get outright
support of New Zealand for its bid for Nuclear Supplier Group (NSG)
membership as New Zealand is a member of NAC.

Q Source: http://www.thehindu.com/news/national/narendra-
modijohn-key-bilateral-talks-india-gets-constructive-promise-from-
new-zealand-on-nsg-bid/article9271397.ece

58. Dhamma mahamattas, in the history of India, were officials


who were appointed to administer
a) Civil and Criminal justice
b) Land deeds attached to religious institutions
c) Translation of vedic texts
d) None of the above

Solution: d)


INSIGHTS PRELIMS TEST SERIES 2017

www.insightsonindia.com www.insightsias.com
TEST 9 Solutions

Learning: Ashoka tried to hold his empire together by propagating


dhamma, the principles of which, as we have seen, were simple and
virtually universally applicable.

This, according to him, would ensure the well-being of people in


this world and the next.
Special officers, known as the dhamma mahamatta, were
appointed to spread the message of dhamma.
Dhamma Policy included included respect towards elders,
generosity towards Brahmanas and those who renounced worldly
life, treating slaves and servants kindly, and respect for religions
and traditions other than ones own.
However, they gradually developed into a type of priesthood of
Dhamma with great powers and soon began to interfere in politics.
Q Source: Page 34: Themes in Indian History I

59. Evidence derived from several Interglacial periods is used


to predict which of the following?
1. Climate change on earth
2. Sea floor spreading
3. Initial loss of atmosphere from earth

Select the correct answer using the codes below.

a) 1 only
b) 1 and 2 only
c) 2 and 3 only
d) 1 and 3 only

Solution: a)
Justification: An interglacial period is a geological interval of warmer
global average temperature lasting thousands of years that separates
consecutive glacial periods within an ice age.

Evidence of multiple advances and retreats of glaciers, and the


sediment deposits in glacial lakes reveal the occurrence of warm


INSIGHTS PRELIMS TEST SERIES 2017

www.insightsonindia.com www.insightsias.com
TEST 9 Solutions

and cold periods in the history of earth. It is a strong evidence of


climate change. Hence (a) is the answer.
The causes of sea floor spreading have been dealt well in the
previous tests. Interglacial periods have no relation with sea floor
spreading, which has to do with volcanism in the mid-oceanic
ridges. So, 2 is incorrect.

Statement 3 is absurd.

Learning: Tree rings also show evidence for wet and dry periods. Also,
for e.g. near Rajasthan, 3000-1700 BC was a period of higher rainfall,
with 2000-1700 BC supporting Harappan civilization. Dry conditions
accentuated since then.

These are used to study phenomenon of climate change in a region.

Q Source: Revision previous test syllabus: Fundamentals of Physical


Geography: 11th NCERT

60. In a mixed economic system


a) Only consumer goods are produced for sale in a general market
b) Profits are shared between government and capitalists
c) State-run enterprises are disinvested once they start making
profits
d) Both the government and the market together decide the
economic production and distribution

Solution: d)
Justification: Option A: India is a mixed economy, and we produce all
kinds of goods, whether consumer goods (e.g. soap), or intermediary
goods (e.g. steel), or capital goods (e.g.machinery). So, A is wrong.

Option B: This is not a general feature of any economic system; however


on an explicit contract in a PPP arrangement, there can be a provision


INSIGHTS PRELIMS TEST SERIES 2017

www.insightsonindia.com www.insightsias.com
TEST 9 Solutions

like this. To that extent it can be a feature of a capitalistic society also.


So, B is incorrect.

Option C: They are disinvested for strategic reasons which may vary
from case to case, such as gaining more efficiency, managerial pool,
capital infusion etc. But, it is not a part of a mixed economy as a general
rule. So, C is incorrect.

Option D: Most economies are mixed economies, i.e., the government


and the market together answer the three questions of what to produce,
how to produce and how to distribute what is produced. In a mixed
economy, the market will provide whatever goods and services it can
produce well, and the government will provide essential goods and
services which the market fails to do.

Q Source: Improvisation: Page 18: Indian Economic Development:


Class XIth NCERT

61.Consider the following about the views of certain travellers about


the social and political aspects of the Indian sub-continent.
1. Megasthenes wrote that the military organization in India was
extremely poor and often left to the control of civilians.
2. Chinese pilgrim Xuan Zang observed that executioners and
scavengers were forced to live outside the city.
3. Fa Xian wrote that untouchables had to sound a clapper in the
streets so that people could avoid seeing them.

Select the correct answer using the codes below.

a) 1 and 2 only
b) 2 and 3 only
c) 1 and 3 only
d) 1, 2 and 3


INSIGHTS PRELIMS TEST SERIES 2017

www.insightsonindia.com www.insightsias.com
TEST 9 Solutions

Solution: b)
Justification: Statement 1: Megasthenes visited the Mauryan Empire
and noted that a committee existed with six subcommittees for
coordinating military activity.

Of these, one subcommittee looked after the navy, the second managed
transport and provisions, the third was responsible for foot-soldiers, the
fourth for horses, the fifth for chariots and the sixth for elephants.
Therefore, statement 1 does not hold ground.

Statement 2 and 3: The condition of the lower classes was very poor in
ancient India. Most such prescriptions were given by Hindu law texts
like Manusmriti. There was least social opposition to such texts in India
then.

Q Source: Chapter 3: Themes in Indian History I

62. In India Foreign exchange reserves are managed


a) By the Reserve Bank of India
b) By the Government of India
c) Jointly by the RBI and Scheduled Commercial Banks
d) Jointly by the RBI and International Monetary Fund (IMF)

Solution: a)
Learning: Forex consist of Gold, SDRs and foreign currency. They are
managed by RBI.

The reserves deplete or acrrue based on the total BoP deficit/surplus of


India. If CAD is too high and capital flows cannot bridge it, forex will be
depleted.

If BoP is in surplus, forex will increase. RBI often intervenes in the


domestic market by sterilizing foreign capital inflows.


INSIGHTS PRELIMS TEST SERIES 2017

www.insightsonindia.com www.insightsias.com
TEST 9 Solutions

For e.g. if India receives FDI and FII in excess of what it needs to bridge
the CAD, the RBI sells rupee in the forex market and buys those foreign
currencies. This leads to increase in the money supply in India.

Q Source: Improvisation: Page :Indian Economic Development: Class


XIth NCERT

63. Recently, the following African countries have been at the


centres of many civil and military conflicts. Which one of these
countries is land locked?
a) Libya
b) Sudan
c) Nigeria
d) Mali

Solution: d)
Justification: You can read about the Mali conflict here
http://www.aljazeera.com/news/2016/07/mali-20-dead-fighting-
threatens-peace-deal-160722210119514.html

Libya was a part of the Arab Spring movement for establishment of


democracy in the middle-east.

Sudan was partitioned into North and South Sudan as a result of coups
and following civil war.

The Boko Haram insurgency in Nigeria began in 2009, when the jihadist
rebel group Boko Haram started an armed rebellion against the
government of Nigeria. The insurgency hasnt subsided completely.


INSIGHTS PRELIMS TEST SERIES 2017

www.insightsonindia.com www.insightsias.com
TEST 9 Solutions

Q Source: Map-based questions: Africa

64. Consider the following major Political and Economic


Developments in the Indian sub-continent.
1. Invasion of Alexander of Macedon
2. Earliest inscriptional evidence of land grants by Satavahana and
Shaka rulers
3. Chinese pilgrim Xuan Zang comes in search of Buddhist texts

The correct chronological order of the above is?

a) 123
b) 213
c) 321
d) 132

Solution: a)


INSIGHTS PRELIMS TEST SERIES 2017

www.insightsonindia.com www.insightsias.com
TEST 9 Solutions

Learning: In 500-400 BCE Rulers of Magadha consolidated power and


in 327-325 BCE Alexander of Macedon invaded the sub-continent.

Accession of Chandragupta Maurya happened in c. 321 BCE.

Earliest inscriptional evidence of land grants by Satavahana and Shaka


rulers in 100-200 CE.

In 606-647 CE Harshavardhana became the king of Kanauj and Chinese


pilgrim Xuan Zang comes in search of Buddhist texts in India.

You can find the entire timeline at the Q Source.

Q Source: Page 50: Themes in Indian History I

65. Bharat net project aims to


a) Connect all households of India through broadband
b) Strengthen existing e-Governance portals in all Panchayat offices
c) Revive sick PSUs related to broadband and communication in
India
d) Link all institutions of higher learning by a common network

Solution: a)
Justification: BharatNet project aims to connect all households of
India, particularly in rural areas, through broadband (2-20 Mbps) by
2017.

It is a remodelled version of National Optical Fibre Network (NOFN)


started in 2011 to connect all 2,50,000 gram panchayats.

Bharat Broadband Network Limited (BBNL), a special purpose vehicle,


has been set up under the telecom ministry for the establishment,
management and operation of NOFN.

BSNL, Railtel and Power Grid are executing agencies for the project.


INSIGHTS PRELIMS TEST SERIES 2017

www.insightsonindia.com www.insightsias.com
TEST 9 Solutions

Learning: TRAI has recently recommended for PPP model in form of


Build-Own-Operate Transfer/Build-Operate-Transfer models for roll out
of Bharat Net project.

Rural broadband provision is prone to market failure as well as


Government failure (delay in implementation of the project)

TRAI wants to include private sector companies in not just deployment


but also implementation of the optic fibre cable (OFC) network.

Q Source: http://www.thehindu.com/business/trai-for-ppp-model-for-
bharat-net-project/article8179848.ece

66. Micro-credit programmes are generally associated with


a) State loans to local bodies
b) Trade cartels
c) Self-Help Groups (SHGs)
d) Scheduled Commercial Banks

Solution: c)
Learning: The innovative idea of Microcredit originated with the
Grameen Bank in Bangladesh. The Grameen Bank is a microfinance
organization and community development bank started in Bangladesh
that makes small loans known as microcredit.

It can be defined as provision of small credit to the poor in rural,


semi-urban and urban areas for enabling them to raise their
income levels and improve living standards.
The institutions that provide Micro Credit are called Micro Credit
Institutions.
Micro Credit is provided to those individuals that lack collateral,
steady employment and a verifiable credit history and therefore
cannot meet even the most minimal qualifications to gain access to
traditional credit.


INSIGHTS PRELIMS TEST SERIES 2017

www.insightsonindia.com www.insightsias.com
TEST 9 Solutions

Sometimes SHGs save among themselves and provide micro-


credit.

Q Source: Improvisation: Page 103: Indian Economic Development:


Class XIth NCERT

67. Consider the following with reference to the Atal Innovation


Mission (AIM).
1. Entrepreneurship promotion is one of the core functions of the
mission.
2. It is a sub-component of the scheme Self-Employment and Talent
Utilization (SETU).
3. The Mission financially supports infrastructural development
carried out in major higher educational and research institutions
in India.
4. Atal Tinkering Laboratories (ATLs) are being established in
schools across India under the mission.

Select the correct answer using the codes below.

a) 2, 3 and 4 only
b) 1 and 4 only
c) 1 and 3 only
d) 1, 2, 3 and 4

Solution: b)
Justification: Statement 2: Atal Innovation Mission (AIM) including
Self-Employment and Talent Utilization (SETU) is Government of
Indias endeavour to promote a culture of innovation and
entrepreneurship. AIM is not a sub-component of SETU. They are
different missions. So, 2 is wrong.

Statement 1: The Atal Innovation Mission shall have two core functions:

Entrepreneurship promotion through Self-Employment and Talent


Utilization, wherein innovators would be supported and mentored
to become successful entrepreneurs


INSIGHTS PRELIMS TEST SERIES 2017

www.insightsonindia.com www.insightsias.com
TEST 9 Solutions

Innovation promotion: to provide a platform where innovative


ideas are generated
Statement 3: Its objective is to serve as a platform for promotion of
world-class Innovation Hubs, Grand Challenges, Start-up businesses and
other self-employment activities, particularly in technology driven areas.
So, 3 is wrong.

Statement 4: With a vision to Cultivate one Million children in India as


Neoteric Innovators, Atal Innovation Mission is establishing Atal
Tinkering Laboratories (ATLs) in schools across India. The objective of
this scheme is to foster curiosity, creativity and imagination in young
minds; and inculcate skills such as design mindset, computational
thinking, adaptive learning, physical computing etc.

Learning: AIM intends to establish new incubation centres (Atal


Incubation Centres) across India by providing them with financial
support. AICs would further support and encourage start-ups to become
successful enterprises. They would provide necessary and adequate
infrastructure along with high quality assistance or services to start-ups
in their early stages of growth.

Atal Grand Challenge (AGC) Awards, under the Atal Innovation


Mission, has the objective of developing novel disruptive
technologies that are ultra-low cost, low maintenance, durable and
customised to the local conditions of India. AGC will award grand
prizes to anyone who delivers in a timely manner the desired
solution as per the challenge specific criteria.
Q Source: Improvisation: NITI Website: From Chapter 2 Mention of
Planning Commission: Indian Economic Development: Class XIth
NCERT

68. The state of Sikkim borders which of the following Indian


states?
1. West Bengal
2. Arunachal Pradesh
3. Assam
4. Uttar Pradesh


INSIGHTS PRELIMS TEST SERIES 2017

www.insightsonindia.com www.insightsias.com
TEST 9 Solutions

Select the correct answer using the codes below.

a) 1 only
b) 1 and 2 only
c) 3 and 4 only
d) 1 and 3 only

Solution: a)
Justification: It touches only West Bengal. It is bordered by China,
Nepal and Bhutan.

Q Source: Improvisation: CAPF past year papers

69. The Union Government is developing a National Data


Registry (NDR) as an important step towards Digital India. Which


INSIGHTS PRELIMS TEST SERIES 2017

www.insightsonindia.com www.insightsias.com
TEST 9 Solutions

of these Ministries/departments will be the nodal agency for the


same?
a) Department of Science and Technology (DST)
b) Ministry of Electronics and Information Technology
c) Ministry of Home Affairs
d) Department of Personnel

Solution: a)
Learning: It will compile and serve metadata of different agencies and
re-engineer the feature data sets for improving their use.

The registry will also serve as a source of authenticated


information. The Department of Science and Technology (DST)
will be the nodal coordinating agency of the NDR.
NDR will require all agencies state, private and academic,
collecting and storing geospatial data to provide details of data
they store.
The registry aims to create a catalogue that will prevent duplication
of data sets and to help users locate the right agencies to source
information.
It will be a meta-data repository to only inform about the nature
of the data a service provider has.
It can operate across a wide range of formats and can source
information automatically. Thus, Survey of India can vet it for
accuracy and see whether it contains information that contravenes
national security.

Q Source: http://www.dst.gov.in/news/rfp-setting-national-data-
registryndr-nsdi-pre-bid-meeting

70. Consider the following about the Man Booker Prize.


1. It can be awarded for literature work in any internationally
recognized language.
2. No person of India origin has won the award till date.

Which of the above is/are correct?


INSIGHTS PRELIMS TEST SERIES 2017

www.insightsonindia.com www.insightsias.com
TEST 9 Solutions

a) 1 only
b) 2 only
c) Both 1 and 2
d) None

Solution: d)
Justification: Statement 1: The Man Booker Prize worlds most
prestigious English-language literary award. The award aims at
promoting the finest literary work in fiction by rewarding the best novel
of the year written in English and published in the United Kingdom.

Statement 2: There have been few Indian (origin) awardees:

For e.g. in 2006- Kiran Desai for The Inheritance of Loss which was a
Novel.

In 2008, Arvind Aadiga was awarded the prize for the White Tiger.

Q Source: http://indiatoday.intoday.in/education/story/5-indians-
who-made-it-to-the-booker-prize/1/430834.html

71. Regional Comprehensive Economic Partnership (RCEP) does NOT


propose to include which of these nations?
a) Bangladesh
b) Thailand
c) Australia
d) India

Solution: a)
Learning: RCEP is the proposed mega-regional Free Trade Agreement
(FTA) between 16 Asia-Pacific countries. It includes India, China, Japan,
South Korea, Australia, New Zealand and the 10-member ASEAN bloc.
The RCEP negotiation process had started in 2013 and is unlikely to be
concluded by the end of 2016.


INSIGHTS PRELIMS TEST SERIES 2017

www.insightsonindia.com www.insightsias.com
TEST 9 Solutions

The proposed FTA aims to open up trade in goods and services as


well as liberalise investment policies.
The 15 Asia-Pacific nations including India of the Regional
Comprehensive Economic Partnership (RCEP) are concerned to
give greater market access to Chinese goods without gaining in
return.
All these countries in the RCEP grouping are also worried about
agreeing to eliminate tariffs altogether, a move that will mainly
help China.
They fear that China may dump its excess capacity in several items
including steel, as well as highly subsidised items which may harm
local industry of importing countries and distort trade.

Q Source: Often in news

72. Russia has invited India to join in developing next-


generation nuclear reactors and to participate in its fast-reactor
research project MBIR. What is purpose of this project?
a) To produce ultra thin reactors that can even be placed
comfortably in urban areas
b) To use gravitational waves as a moderator in nuclear reactions
c) To simulate fusion reactions
d) To enable the recycling the nuclear waste as new fuel

Solution: d)
Learning: The multipurpose fast research reactor project known by
Russian acronym MBIR is coming up as the International Research
Centre (IRC) in Russia.

It focuses on creation of a new technological platform for nuclear energy,


which will be based on the closed fuel cycle with fast neutron reactors

The MBIR project is based on the closed fuel cycle which involves
recycling the nuclear waste as new fuel.


INSIGHTS PRELIMS TEST SERIES 2017

www.insightsonindia.com www.insightsias.com
TEST 9 Solutions

It essentially signifies research on a sodium-cooled Generation 4 fast


reactor to design an advanced fast neutron reactor for use in nuclear
power plants. MBIRs design includes three independent loops that can
be used to test different coolants like gas, lead, molten salt, among
others. Therefore it makes possible to conduct material testing research
in those different environments

Q Source: http://www.thehindu.com/business/Industry/russia-
invites-india-to-join-fast-reactor-research-project/article9288233.ece

73. Consider the following about Indian Systems of Medicine


(ISM).
1. It includes Ayurveda, Siddha and Unani systems among others.
2. All the systems have developed indigenously in India.
3. These systems have not been official recognized for lack of
authoritative material.

Select the correct answer using the codes below.

a) 2 only
b) 1 and 3 only
c) 1 only
d) 1, 2 and 3

Solution: c)
Justification: Statement 1: The Indian Systems of Medicine and
Homoeopathy consist of Ayurveda, Siddha, Unani and Homoeopathy,
and therapies such as Yoga and Naturopathy.

Statement 2: Some of these systems are indigenous and others have over
the years become a part of Indian tradition such as Unani.

Statement 3: There is a dedicated department of AYUSH in the Central


government and many specific departments in state governments that
deal with ISM.


INSIGHTS PRELIMS TEST SERIES 2017

www.insightsonindia.com www.insightsias.com
TEST 9 Solutions

You can read briefly about them here


http://www.ism.kerala.gov.in/index.php/about-ism.html

Q Source: Improvisation: Page 153: Indian Economic Development:


Class XIth NCERT

74. These are some of the joint military exercises India holds
with several countries. Consider the matches of exercises with the
partner nations.
1. Indradhanush : India-UK
2. Varuna : India-Sri Lanka
3. Exercise Malabar: India-USA
4. Surya Kiran : India-Nepal

Select the correct answer using the codes below.

a) 1 and 3 only
b) 2 and 4 only
c) 1, 3 and 4 only
d) 1, 2, 3 and 4

Solution: c)
Justification: These are frequently in news. Here is a useful
compilation.

Indian Navy Joint Exercises

Varuna naval exercise : France and India.


SLINEX-Sri Lanka India
INDRA - India and Russia
Exercise Malabar - United States and India.
Simbex - Indian Navy with Republic of Singapore Navy
IBSAMAR with the Brazil and South African navies.
KONKAN - A bilateral Naval Exercise between Indian Navy and
Royal Navy of Britain


INSIGHTS PRELIMS TEST SERIES 2017

www.insightsonindia.com www.insightsias.com
TEST 9 Solutions

Indian Army Joint Exercises

Mithra Shakti exercise- India and Sri Lanka.



Hand-in-Hand -India-China joint military training exercise

Exercise Shakti - India and France armies

Exercise Nomadic Elephant- Indian Army exercises with the

Mongolian Army.
Exercise Yuddh Abhyas is a series of joint exercises between the
Indian and United States Armies since 2005.
Surya Kiran - India Nepal joint military exercise
LAMITYE- India and Seychelles
Prabal Dostyk - India Kazhakhstan Joint exercise
Q Source: Frequently in news: For example see,
www.pib.nic.in/newsite/PrintRelease.aspx?relid=118561

http://www.ndtv.com/india-news/indo-nepal-joint-military-exercise-
kicks-off-1587086

75. The term Navaratnas is generally used for which of the


following?
1. Group of scholars in the Imperial Court of King Vikramaditya
and Akbar
2. Select group of Public Sector Enterprises (PSUs)
3. Profit making Central departmental enterprises (CDEs)
4. Group of Kingdoms that occupied the North-Western province of
India before the invasion of Ghazni

Select the correct answer using the codes below.

a) 2 only
b) 3 and 4 only
c) 1, 2 and 4 only
d) 1 and 2 only

Solution: d)
Justification: There was no such group as mentioned in statement 4.


INSIGHTS PRELIMS TEST SERIES 2017

www.insightsonindia.com www.insightsias.com
TEST 9 Solutions

All profit making enterprises are not a part of the navratna group. So, 3
is incorrect.

Learning: In 1996, in order to improve efficiency, infuse


professionalism and enable them to compete more effectively in the
liberalised global environment, the government chose nine PSUs and
declared them as navaratnas.

They were given greater managerial and operational autonomy, in


taking various decisions to run the company efficiently and thus
increase their profits.
Greater operational, financial and managerial autonomy had also
been granted to 97 other profit-making enterprises referred to as
mini ratnas.
Nine gems during the reign of Vikramaditya (Chandragupta II) of
the Gupta Empire were: Kalidasa, Vetala Bhatta, Varahamihira,
Vararuchi, Amarasimha, Dhanvantari, Kahapanak, Shanku,
Harisena .
Nine courtiers of Emperor Akbar or Navratnas were: Birbal, Faizi,
Todar Mal, Man Singh I, Abdul Rahim Khan-I-Khana, Fakir Aziao-
Din, Tansen, Mulla Do-Piyaza, Abu'l-Fazl ibn Mubarak.
Q Source: Page 44: Indian Economic Development: Class XIth NCERT

76. Which of the following states of India has been officially


declared defecation-free?
a) Himachal Pradesh
b) Kerala
c) Sikkim
d) All of the above

Solution: d)


INSIGHTS PRELIMS TEST SERIES 2017

www.insightsonindia.com www.insightsias.com
TEST 9 Solutions

Learning: Open Defecation often leads to various diseases like


intestinal worm infections, diarrhoea, polio, hepatitis etc. It is a major
policy goal to bring all states in India to this status.

Every individual household has functional toilet under the Swachh


Bharat Mission in all these three states. Kerala and HP have achieved
the feat recently. Sikkim was the first state.

This status will further entitle Himachal Pradesh to receive World Bank
funding under Rs 9,000 crore project to sustain sanitation campaign.

Q Source: In news recently

77.Consider the following about BrahMos Missile.


1. It can be fired from land, sea and air.
2. It is a supersonic cruise missile.
3. It uses atmospheric oxygen as a fuel.
4. It was inducted in the Indian armed forces recently only after
India became a member of the Missile Technology Control
Regime (MTCR).

Select the correct answer using the codes below.

a) 1, 2 and 3 only
b) 2, 3 and 4 only
c) 1 and 2 only
d) 3 and 4 only

Solution: c)
Justification: Statement 1: It operates on fire and forget principal and
is capable of being launched from land, sea, sub-sea and air against sea
and land targets.

Statement 2: It is capable of carrying a warhead and has top supersonic


speed of Mach 3 (that is, three times the speed of sound).


INSIGHTS PRELIMS TEST SERIES 2017

www.insightsonindia.com www.insightsias.com
TEST 9 Solutions

Statement 3: It is two-stage missile, the first one being solid and the
second one ramjet liquid propellant.

Statement 4: The BrahMos Missile already been deployed by Indian


Army and Indian Navy in anti-ship and precision strike roles
respectively. The air version is at present undergoing testing.

However, Russia was unable to supply crucial updates to the Brahmos to


increase it range due to non-member restrictions for India under the
MTCR regime. Now that India has become a MTCR member, Russia has
supplied these crucial inputs to India.

Q Source: http://www.thehindu.com/news/national/range-of-
brahmos-to-be-doubled/article9282470.ece

78. Which of these can be categorized as capital goods


industries?
1. Engineering goods
2. Textiles
3. Financial Services Industry
4. Construction and mining equipment industry

Select the correct answer using the codes below.

a) 1 and 4 only
b) 2 and 3 only
c) 1, 2 and 3 only
d) 2 and 4 only

Solution: a)
Justification: Capital Goods refer to products that are used in the
production of other products but are not incorporated into the new
product. These include machine tools, industrial machinery, process
plant equipment, construction & mining equipment, electrical
equipment, textile machinery, printing & packaging machinery etc.


INSIGHTS PRELIMS TEST SERIES 2017

www.insightsonindia.com www.insightsias.com
TEST 9 Solutions

They are deployed to help increase future production. Consumer goods


are any goods that are not capital goods; they are goods used by
consumers and have no future productive use.

Statement 2: They make consumer goods, hence wrong.

Statement 3: They produce services, which are essentially consumed, or


may be used further in the economic process, but they are not classified
as capital goods industries as such. So, 3 is wrong.

Q Source: Improvisation: Page 7:Indian Economic Development: Class


XIth NCERT

79. Equator passes through which of these African countries?


1. Congo
2. Kenya
3. Niger
4. Namibia

Select the correct answer using the codes below.

a) 1 and 2 only
b) 2 and 3 only
c) 1 and 4 only
d) 3 and 4 only

Solution: a)
Justification: Africa is the only continent through which all the major
latitudinal lines pass - Tropic of Cancer, Tropic of Capricorn and
Equator.

Equator runs through Gabon, Republic of Congo, Democratic Republic


of Congo, Uganda, Kenya and Somalia.


INSIGHTS PRELIMS TEST SERIES 2017

www.insightsonindia.com www.insightsias.com
TEST 9 Solutions

Tropic of Cancer passes through Mali, Algeria, Niger, Libya, Egypt and
Mauritania.

The Tropic of Capricorn passes through the countries of Namibia,


Botswana, South Africa and Mozambique.

Q Source: Map-based questions: Africa

80. The Union Cabinet has recently given its in-principle


approval for strategic sale of many public sector undertakings
(PSUs). What do you understand by strategic sale?
a) Transfer of management and significant ownership from
government to the private sector
b) Transfer of key state-of-the-art technologies from government to
private sector
c) Transfer of administrative staff only to the private sector
d) Transfer of all shares of PSUs to retail investors

Solution: a)
Learning: In Strategic disinvestment the management control and a
significant proportion of a PSUs share goes to a private sector strategic
partner.


INSIGHTS PRELIMS TEST SERIES 2017

www.insightsonindia.com www.insightsias.com
TEST 9 Solutions

Thus, strategic disinvestment of a PSU is different from the ordinary


disinvestment in which management of PSU is retained with
Government.

According to the Department of Disinvestment, in the strategic


disinvestment of a PSU, the transaction has two elements: (i) Transfer of
a block of shares to a Strategic Partner and (ii) Transfer of management
control to the Strategic Partner.

Q Source: PIB Cabinet decisions

81. Gross Domestic Product (GDP) is often calculated based on


the value added method. What do you understand by the term
value added?
a) Difference in the costs of input and output
b) Difference in market price and subsidized price
c) Difference in export price and import parity price
d) Difference between earnings and taxes on the product

Solution: a)
Learning: In the product method, the total market value of all FINAL
goods and services produced within an economy in a financial year is
added up. Intermediary goods and services are discounted from the
calculation to avoid "double counting".

The difference between the final goods and the value of


intermediary inputs used at each stage of production is called the
"value added". When the value added by all goods and services are
added up, it gives us the final GDP.
Gross value added adjusted for depreciation, and indirect taxes
(and subsidies) becomes the net domestic product at factor cost, or
the National Income.


INSIGHTS PRELIMS TEST SERIES 2017

www.insightsonindia.com www.insightsias.com
TEST 9 Solutions

Value added estimate is useful to track sectoral growth trends in


the economy; i.e. whether agriculture is contributing more to
growth, or if industry is doing so.
Q Source: Improvisation: Page 8:Indian Economic Development: Class
XIth NCERT

82. The Living Planet Report is published by


a) United Nations Environment Programme (UNEP)
b) World Wildlife Fund (WWF)
c) Greenpeace
d) Convention on Biological Diversity (CBD) Secretariat

Solution: b)
Learning: According to a recently released study, global wildlife
populations have fallen by 58% since 1970 and if the trend continues
then two-thirds of wild animals may go extinct by 2020.

The study was published as The Living Planet assessment by the


Zoological Society of London (ZSL) and the World Wildlife Fund
(WWF).
It suggests that animals living in lakes, rivers and wetlands are
suffering the biggest losses.
It also concluded that vertebrate populations are declining by an
average of 2% each year.
The Living Planet Report is published every two years. It aims to
provide an assessment of the state of the worlds wildlife. The last
report was published in 2014. It had estimated that the worlds
wildlife populations had halved over the last 40 years.
Q Source: http://www.thehindu.com/todays-paper/tp-
international/twothirds-of-wild-animals-may-go-extinct-by-2020-
warns-report/article9278553.ece


INSIGHTS PRELIMS TEST SERIES 2017

www.insightsonindia.com www.insightsias.com
TEST 9 Solutions

83. Some of the longer-term results of global warming can be


1. Coastal flooding
2. Increased incidence of tropical diseases
3. Disappearance of some ecological niches
4. Disruption of drinking water supplies
5. Increased frequency of tropical storms

Select the correct answer using the codes below.

a) 1, 3 and 5 only
b) 2, 4 and 5 only
c) 1, 3 and 4 only
d) 1, 2, 3, 4 and 5

Solution: d)
Justification: Statement 1: This occurs due to mean sea level rise as
oceans get warmer and water expands due to global warming.

Statement 2: Tropical Disease vectors thrive in higher temperature


climates. As temperature rises due to global warming, their survival
becomes more established.

Statement 3: Global climate change has already had observable effects


on the environment. Glaciers have shrunk, ice on rivers and lakes is
breaking up earlier, plant and animal ranges have shifted and trees are
flowering sooner. Species that thrive only in a particular ecosystem i.e.
niche (lower tolerance) will decline sooner.

Statement 4: Disruption of drinking water supplies will happen as


glaciers disappear that give rise to many perennial rivers.

You can read here http://climate.nasa.gov/effects/

https://www2.usgs.gov/faq/categories/9772/5641

Q Source: Improvisation: Page 165: Indian Economic Development:


Class XIth NCERT


INSIGHTS PRELIMS TEST SERIES 2017

www.insightsonindia.com www.insightsias.com
TEST 9 Solutions

84. Given below are some of the earliest states matched with
their capitals. Suggest correct matches among them.
1. Malla : Kusinagara
2. Vatsa : Kaushambi
3. Anga : Vanga
4. Matsya : Ashmaka

Select the correct answer using the codes below.

a) 1 and 2 only
b) 2 and 4 only
c) 1 and 3 only
d) 1 and 4 only

Solution: a)
Justification: Statement 1: Its capital was Kushinagar, Pawa. It
covered the modern districts of Deoria, Basti, Gorakhpur in Eastern
Uttar Pradesh.

Statement 2: Its capital was Kausambi. It covered the modern districts


of Allahabad and Mirzapur in Uttar Pradesh.

Statement 3: Champa was the capital of Anga.

Statement 4: Matsya were one of the Indo-Aryan tribes of Vedic India.


By the late Vedic period, they ruled a kingdom located south of the
Kurus, and west of the Yamuna River which separated it from the
kingdom of the Panchalas. It roughly corresponded to the former state of
Jaipur in Rajasthan. The capital of Matsya was at Viratanagari (present-
day Bairat). Alwar has been a part of Matsya region.


INSIGHTS PRELIMS TEST SERIES 2017

www.insightsonindia.com www.insightsias.com
TEST 9 Solutions

Q Source: Page 30: Themes in Indian History I

85. What is often referred to as the Balance of payments crisis


is actually
a) A potential national default on its external obligations
b) A situation when government is unable to finance its fiscal deficit
c) A high percentage of revenue expenditure in the government
budget
d) A zero current account deficit

Solution: a)
Learning: The Balance of Payments shows a countries transactions
with the rest of the world.


INSIGHTS PRELIMS TEST SERIES 2017

www.insightsonindia.com www.insightsias.com
TEST 9 Solutions

The current account comprises the trade balance (which is trade in


goods) and also includes the balance for trade in services.

When people refer to a balance of payments deficit they invariably mean


a current account deficit. But, this is not the case as the BoP also includes
the capital account, which includes FDI, FII, external long-term debt etc.
So, D cant be the answer.

Balance of Payments Crisis occurs when the current account deficit


cannot be maintained. It means there will be a fall in foreign exchange
reserves and the country can no longer attract sufficient capital flows to
finance the current account deficit. India faced the BoP crisis in 1991.

Q Source: Improvisation: Page 40: Indian Economic Development:


Class XIth NCERT

86. Which of the following statements about the Enforcement


Directorate (ED) is INCORRECT?
a) ED is appointed on the approval of the Appointments Committee
of the Cabinet (ACC) headed by Prime Minister.
b) It functions under the aegis of the Central Bureau of
Investigation (CBI).
c) It is responsible for tracking black money and hawala trade cases.
d) None of the above

Solution: b)
Learning: The Union Government has appointed Senior IPS officer
Karnal Singh as the Director of Enforcement Directorate (ED).

The Economic Enforcement is economic intelligence and law


enforcement agency responsible for enforcing economic laws and
fighting economic crime in India.
It functions under the aegis of the Department of Revenue, Union
Ministry of Finance. So, B is incorrect.


INSIGHTS PRELIMS TEST SERIES 2017

www.insightsonindia.com www.insightsias.com
TEST 9 Solutions

It has a mandate to enforce two of most stringent laws in the


country. They are Prevention of Money Laundering Act (PMLA)
and Foreign Exchange Management Act (FEMA) to check black
money and hawala trade cases.
Q Source: PIB Appointments

87. Consider the following statements with reference to the


International Monetary Fund (IMF).
1. It is governed by the International Monetary and Financial
Committee.
2. One of the aims of IMF is to promote exchange stability.
3. India is the biggest borrower from IMF.

Select the correct answer using the codes below.

a) 1 and 2 only
b) 2 and 3 only
c) 1 and 3 only
d) 2 only

Solution: d)
Justification: Statement 1: The IMFC advises and reports to the IMF
Board of Governors on the supervision and management of the
international monetary and financial system, including on responses to
unfolding events that may disrupt the system.

Statement 2: Original aims of IMF:

promote international monetary cooperation;


facilitate the expansion and balanced growth of international
trade;
promote exchange stability;
assist in the establishment of a multilateral system of payments;
and


INSIGHTS PRELIMS TEST SERIES 2017

www.insightsonindia.com www.insightsias.com
TEST 9 Solutions

make resources available (with adequate safeguards) to members


experiencing balance of payments difficulties.
Statement 3: Biggest borrowers from IMF (amounts outstanding as of
2015 first quarter): Portugal, Greece, Ireland, Ukraine.

Q Source: Improvisation: Page 40: Indian Economic Development:


Class XIth NCERT

88. Consider the following about the Comprehensive


Environmental Pollution Index (CEPI).
1. It has been developed by the Central Pollution Control Board
(CPCB).
2. The Index is a rational number to characterize the
environmental quality at a given location.
3. The index is used to earmark buffer zones around protected areas
using different colour schemes.

Select the correct answer using the codes below.

a) 1 only
b) 1 and 2 only
c) 2 and 3 only
d) 1, 2 and 3

Solution: b)
Justification: The index captures the various health dimensions of
environment including air, water and land.

The present CEPI is intended to act as an early warning tool and helps in
categorizing the industrial clusters/areas in terms of priority of planning
needs for interventions.

The CEPI is based on parameters such as the nature of toxins, the scale
of industrial activities, the level of ambient pollution, impact on humans


INSIGHTS PRELIMS TEST SERIES 2017

www.insightsonindia.com www.insightsias.com
TEST 9 Solutions

and eco-geological features and the number of people potentially


affected within a 2 km radius.

http://articles.economictimes.indiatimes.com/2015-04-
11/news/61041378_1_water-pollution-pollution-levels-industrial-
clusters

Q Source: CPCB Website and Current Affairs: Taken from Page 170:
Indian Economic Development: Class XIth NCERT

89. Consider the following statements.


1. This planet is known as one of "Ice Giants.
2. It rotates on its axis in a direction opposite to that of earth.
3. The planet hosts natural satellites.

The above refers to?

a) Neptune
b) Venus
c) Uranus
d) Jupiter

Solution: c)
Learning: An ice giant is a giant planet composed mainly of elements
heavier than hydrogen and helium, such as oxygen, carbon, nitrogen,
and sulphur. There are two known ice giants in the Solar System, Uranus
and Neptune.

Uranus has five major moons: Miranda, Ariel, Umbriel, Titania, and
Oberon. Some more have been discovered recently (hence the question).

Every planet in our solar system except for Venus and Uranus rotates
counter-clockwise as seen from above the North Pole; that is from west
to east.

Q Source: News of undiscovered moons orbiting near Uranus


INSIGHTS PRELIMS TEST SERIES 2017

www.insightsonindia.com www.insightsias.com
TEST 9 Solutions

90. Apart from HIV AIDS, diseases that can be transmitted by


sharing used needles are
1. Diarrhoea
2. Chickengunya
3. Hepatitis C

Select the correct answer using the codes below.

a) 1 and 2 only
b) 2 and 3 only
c) 3 only
d) None of the above

Solution: c)
Justification: Blood borne infections are commonly spread by people
who inject drugs and share needles or syringes. Diseases that are spread
this way include: HIV/AIDS, hepatitis B, and hepatitis C.

People who inject drugs can spread viruses and diseases by sharing
needles or drug preparation equipment that has been contaminated with
infected blood.

Viruses and diseases can be spread from person to person even if


someone only tried injecting drugs many years ago.

Q Source: Page 63: 12th Biology NCERT

91.Consider the following physical features of Africa.


1. Atlas mountains
2. Sahara desert
3. Kalahari desert
4. Lake Victoria.
5. Zambezi River

The correct order of these features north-to-south is?


INSIGHTS PRELIMS TEST SERIES 2017

www.insightsonindia.com www.insightsias.com
TEST 9 Solutions

a) 12345
b) 12453
c) 34125
d) 42135

Solution: b)
Justification:

Q Source: Map-based questions: Africa

92. Consider the following statements.


1. James Prinsep is credited with deciphering Brahmi and
Kharosthi scripts.
2. Wheeler is credited with deciphering Meluha script.

Which of the above is/are correct?


INSIGHTS PRELIMS TEST SERIES 2017

www.insightsonindia.com www.insightsias.com
TEST 9 Solutions

a) 1 only
b) 2 only
c) Both 1 and 2
d) None

Solution: a)
Learning: Most Asokan inscriptions were in the Prakrit language while
those in the northwest of the subcontinet were in Aramaic and Greek.

Most Prakrit inscriptions were written in the Brahmi script; however,


some, in the northwest, were written in Kharosthi.

James Prinsep, an officer in the mint of the East India Company,


deciphered Brahmi and Kharosthi, two scripts used in the earliest
inscriptions and coins. This gave a new direction to investigations into
early Indian political history.

From the late eighteenth century, European scholars aided by Indian


pundits worked backwards from contemporary Bengali and Devanagari
manuscripts, comparing their letters with older specimens. James
Princep was able to decipher Asokan Brahmi in 1838.

Q Source: Page 28: Themes in Indian History I

93. Gondwana system of rocks contains major coal deposits of


India. They can be found in
1. Damodar Valley
2. Son Valley
3. Rajmahal Hills

Select the correct answer using the codes below.

a) 1 and 2 only
b) 1 only
c) 2 and 3 only
d) 1, 2 and 3

Solution: d)


INSIGHTS PRELIMS TEST SERIES 2017

www.insightsonindia.com www.insightsias.com
TEST 9 Solutions

Justification: The important coal bearing areas of this series are


Raniganj, Jharia, Karanpur, and Bokaro of the Damodar basin in Odisha,
and the Pench valley in Chhattisgarh and Madhya Pradesh, the jhingurda
coal seam (Chhattisgarh).

The Gondwana Supergroup forms a unique sequence of fluviatile rocks


deposited in Permo-Carboniferous time.

Damodar and Sone river valley and Rajmahal hills in the eastern India is
depository of the Gondwana rocks.

Q Source: Improvisation: CAPF past year papers


INSIGHTS PRELIMS TEST SERIES 2017

www.insightsonindia.com www.insightsias.com
TEST 9 Solutions

94. The Jarewas, Onges, Sentelenese and Great Andamanis


tribes belong to which racial group?
a) Negritoes
b) Proto-Australoids
c) Mongoloids
d) Western Brachycephals

Solution: a)
Justification:The Negritos or the brachycephalic (broad headed) from
Africa were the earliest people to inhabit India. They are survived in
their original habitat in the Andaman and Nicobar Islands.

Learning: Pro-Australoids or Austrics group was the next to come to


India after the Negritos.

The Austrics were the main builders of the Indus Valley Civilisation.
They cultivated rice and vegetables and made sugar from sugarcane.
Their language has survived in the Kol or Mundan(Mundari) in Eastern
and Central India.

Q Source: Tribes of India

95. Mahatma Gandhi was opposed to separate electorates for


untouchables for which of the following reasons?
1. The system did not allow other communities to get reserved seats
in legislatures.
2. A leader could get elected only once on a separate electorate
ticket.

Which of the above is/are correct?

a) 1 only
b) 2 only
c) Both 1 and 2
d) None

Solution: d)


INSIGHTS PRELIMS TEST SERIES 2017

www.insightsonindia.com www.insightsias.com
TEST 9 Solutions

Justification: Dr. Ambedkar proposed a separate electorate or


reservation of seats for the Untouchables within the Hindu bloc in the
provincial legislatures.

Gandhi opposed the proposal, saying that it would divide the


Hindus and the Harijans. The British saw another opportunity to
divide Indians further and to keep them quarreling among
themselves.
Back in 1909, the British had introduced separate electorates for
the Hindus and the Muslims. As a result, a Hindu could vote only
for a Hindu candidate, a Muslim only for a Muslim.
Consequently, religious differences surfaced in every election, and
for the two communitiesunable to vote for the each others
candidatesthe gulf of mistrust deepened and widened.
So, he was opposed to separate electorates as he saw three Indias:
Hindus, Muslims, and Untouchables going by this system.

Q Source: Improvisation: CAPF past year papers

96. Which of the following developments of the 1860s have a


boost to the Indian industries?
1. Policy of discriminating protection as formally adopted by the
British
2. American Civil War
3. Progressive Ruralization movement

Select the correct answer using the codes below.

a) 1 and 2 only
b) 2 only
c) 2 and 3 only
d) 1 and 3 only

Solution: b)


INSIGHTS PRELIMS TEST SERIES 2017

www.insightsonindia.com www.insightsias.com
TEST 9 Solutions

Justification: Statement 1: The policy was adopted only later in the


1920s in India, which actually worsened the conditions of the Indian
industries. We will explore this topic in greater detail later.

Statement 2: In the 1860s, cotton was a key import, supplying Britains


busy textile mills. The majority of raw cotton supplies came from
America. However, during the American Civil War (186165) raw cotton
was no longer being grown and shipped. The British needed to find new
sources for their mills, so turned to India to supply their needs.

Statement 3: It was actually a movement that worsened the economic


prospects in India. When artisans lost job in mughal courts due to
British policies, they turned to agriculture, a movement which is called
as progressive ruralisation.

Q Source: Improvisation: CAPF past year papers

97. The northern highway known as Uttarapatha in Gangetic


valley connected some of the great cities of ancient India, which
were
a) Kausambi, Prayaga and Kasi
b) Vaisali and Pataliputra
c) Taxila, Mathura and Ahichhatra
d) All of the above

Solution: d)
Learning: There were many major trade routes in India.

The Silk route was a major trading pathway through the first
millennium B.C.
o It connected the kingdom of Kamboja; todays Afghanistan
and Tajikistan, to ancient Pratishthana; Paithan on Godavari
towards south, cities and cultural centers in north India upto
Tamralipti or Tamluk on the eastern sea coast.


INSIGHTS PRELIMS TEST SERIES 2017

www.insightsonindia.com www.insightsias.com
TEST 9 Solutions

As regard the northern part of India, the northern highway known


as Uttarapatha in Gangetic valley was connecting the great cities of
ancient India, which were Taxila, Mathura, Ahichhatra, Sravasti,
Saketa, Kausambi, Prayaga, Kasi, Kusinagara, Vaisali, Pataliputra,
Rajgreha, Bodhagaya and Tamralipti etc.
o The importance of this sub-continental route in the Mauryan
period is evident from the fact that Chandragupta Maurya
from Magadh is supposed to have reached the Indus River
where he met Seleucus the Greek envoy in 305 B.C.
Simultaneously another great trade route of India was
Dakshinapatha. The term Dakshinapatha occurs in the sense of a
trade route to the south.
o Buddhist literatures also note a number of merchants going
from Pataliputra and Kausambi to Pratishthana on the
Godavari.
o The presence of Painted Grey Ware (PGW) and other ceramic
assemblage at a number of sites in the Morena, Ujjain,
Malwa Plateau is indicative of cultural elements travelling
from Northern India to this region.

Q Source: Improvisation: CAPF past year papers

98. Gender Development Index released by the United Nations


Development Programme (UNDP) is
a) The ratio of Female HDI and Male HDI
b) Inequality adjusted Gini index for females
c) A joint index of Labour force participation, reproductive health
and political participation of women
d) None of the above

Solution: a)
Learning: GDI is a relatively new index.


INSIGHTS PRELIMS TEST SERIES 2017

www.insightsonindia.com www.insightsias.com
TEST 9 Solutions

It measures gender gaps in human development achievements by


accounting for disparities between women and men in three basic
dimensions of human development - health, knowledge and living
standards using the same component indicators as in the HDI.

The GDI is the ratio of the HDIs calculated separately for females and
males using the same methodology as in the HDI.

You can read more here

http://hdr.undp.org/en/content/gender-inequality-index-gii

Q Source: Often in news

99. Consider the following about the International Bank for


Reconstruction and Development (IBRD).
1. It was created to help Europe rebuild after the Second World
War.
2. It is owned jointly by the United States and European Union.
3. It supports long-term human and social development that private
creditors may not finance.
4. It provides advisory services in public debt and asset
management to help governments.

Select the correct answer using the codes below.

a) 1 and 2 only
b) 1 and 3 only
c) 3 and 4 only
d) 1, 3 and 4 only

Solution: d)
Justification: Statement 1: It was created in 1944 to help Europe
rebuild after World War II. Today, IBRD provides loans and other
assistance primarily to middle income countries.


INSIGHTS PRELIMS TEST SERIES 2017

www.insightsonindia.com www.insightsias.com
TEST 9 Solutions

Statement 2: IBRD is owned by the governments of its 189 member


countries, not by US and EU alone.

Statement 3: IBRD is the original World Bank institution. It works


closely with the rest of the World Bank Group to help developing
countries reduce poverty, promote economic growth, and build
prosperity.

Specifically, IBRD (apart from statement 3, which is correct):

Preserves borrowers' financial strength by providing support in


times of crisis, when poor people are most adversely affected
Promotes key policy and institutional reforms (such as safety net
or anti-corruption reforms)
Statement 4: IBRDs financial products and services help countries build
resilience to shocks by facilitating access to products that mitigate the
negative impact of currency, interest rate, and commodity price
volatility, natural disasters and extreme weather.

Q Source: Improvisation: Page 40:Indian Economic Development:


Class XIth NCERT

100. A lot of expenditures in the annual budget are charged on the


Consolidated Fund of India. The purpose behind this is to
a) Increase the efficiency of spending
b) Prevent March Rush
c) Reduce the expenditure overlap between several departments
d) None of the above

Solution: d)
Learning: According to Article 112 and Article 202 of the Constitution
of India, the following expenditure does not require a vote and is charged
to the Consolidated Fund.


INSIGHTS PRELIMS TEST SERIES 2017

www.insightsonindia.com www.insightsias.com
TEST 9 Solutions

They include salary, allowances and pension for the President as well as
Governors of States, Speaker and Deputy Speaker of the House of
People, the Comptroller General of India and Judges of the Supreme and
High Courts.

They also include interest and other debt related charges of the
Government and any sums required to satisfy any court judgment
pertaining to the Government.

The idea is to:

Maintain independence of some important offices like judiciary,


since Parliament cannot reduce the expenditure on judges salary
Keep compulsory payments out of the control of legislature (or the
government)
Maintain the dignity of High constitutional offices like President,
because Parliament isnt allowed to vote on these expenditures

Q Source: Improvisation: CAPF past year papers


INSIGHTS PRELIMS TEST SERIES 2017

Das könnte Ihnen auch gefallen